Тема: О силе воздействия на штангу при толчке от груди
Автор: VadimPro
Дата: 15/03/2004 12:47
 	
Данная заметка является продолжением начатой на форуме 
Мовлади Абдулаева дискуссии о силах, придающих штанге 
скорость относительно тела атлета во время толчка от 
груди, а также о роли силы рук и вообще жимовых мышц в 
данном движении. Здесь эта заметка публикуется потому, 
что, во-первых, форум Мовлади Абдулаева позволяет 
размещать сообщения только очень небольшого объёма, а 
во-вторых, потому, что тема этой заметки напрямую 
касается содержания текстов "О быстроте разгибания рук 
при толчке от груди", "Ещё раз о быстроте разгибания 
рук при толчке от груди" и "На Алборова не обижаются", 
размещённых на сайте, к которому относится данный 
форум.

В моей заметке будут рассмотрены кинограммы толчка 
Стефаном Ботевым штанги весом 250 кг на Олимпиаде-96 
в Атланте, размещённые в интернете по следующим
адресам:

Серия 1:
http://www.oswlfan.narod.ru/botevjerk/botevjerk.html

Серия 2:
http://www.oswlfan.narod.ru/botevjerk2/botevjerk2.html

Графики к серии кадров 2:
http://www.oswlfan.narod.ru/botevjerk2/botevgraphs2.htm
l

Указанные кинограммы любезно предоставлены участником
дискуссии Оппонентом, за что ему особая благодарность. 
Без этих материалов точный анализ сил, действующих на 
штангу во время толчка, был бы крайне затруднён, 
излишне теоретизирован и, скорее всего, далёк от 
реальной картины.

Вместе с тем я не могу не отметить, что либо самим
Оппонентом, либо авторами исследования была допущена
ошибка в сопоставлении кадров из серии 2 и графиков
зависимости скорости от пройденного штангой пути.
Под графиком 1 к серии кадров 2 написано, что штанга
находится на 2 см ниже нулевой отметки, однако на
кадре 1 атлет стоит на носках и гриф не прогнут под
весом дисков, то есть диски должны находиться тут 
выше нулевой отметки на величину прогиба грифа и 
подъёма атлета на носки. Собственно, и сам кадр 1 
серии 2 соответствует кадру 4 из серии 1, где в 
комментариях к кадру 4 написано, что это момент отрыва 
штанги от плеч, и данный кадр следует уже за кадром
прохождения нулевой отметки. Таким образом, повторяю, 
штанга на кадре 4 никак не может быть ниже нулевого 
уровня. Судя по всему, в серии 1 совсем нет того 
кадра, который можно было бы сопоставить с графиком 1, 
а кадру 1 соответствует график 2, где штанга имеет 
максимальную скорость 1,81 м/с и находится при этом на 
6 см выше нулевого уровня (согласно подписям под 
графиком). Мои собственные сравнения по числу пикселей 
и сопоставление с диаметром дисков кадра 1 из серии 2 
и кадра 1 из серии 1, обозначенного Оппонентом 
как "Стартовое положение (нулевая точка)", показали, 
что диски штанги на кадре 1 серии 2 находятся выше 
нулевого уровня на 6 см. Кроме того, судя по подписям 
под графиками, на графике 8 штанга находится в 
наивысшей точке в 30 см, а на графике 7 и 9 штанга 
находится ниже на 1 см - однако на кадрах толчка 
штанга находится в наивысшей точке на графике 7. Таким 
образом, нет сомнений, что между графиками и 
раскадровкой произошло смещение на 1 кадр. Итак, кадру 
1 соответствует график 2 и так далее - в том числе 
кадру 7 соответствует график 8.

Поскольку, как утверждает Оппонент, датчик, по
показаниям которого регистрировались параметры подъёма,
находился на торце грифа, измеренная и отложенная на
графиках скорость достаточно близка к скорости
движения дисков - ведь в них, как известно, и 
сосредоточена основная масса штанги. Согласуются с 
этим и сопоставления кадров завершения Ботевым подъёма 
снаряда от груди. На графике отмечено опускание штанги 
примерно на 7 см после достижения максимальной высоты 
подлёта - но на финальных кадрах видно, что гриф 
практически не меняет своего положения после 
достижения максимальной высоты подлёта, а после потери 
скорости под действием силы тяжести опускаются только 
диски штанги. Из этого тоже можно сделать вывод, что 
скорость и перемещение штанги Ботева измерялись именно 
по концам штанги - а не по центру грифа, как это 
практиковалось когда-то советскими исследователями 
Р.Романом и М.Шакирзяновым при регистрации параметров 
движения штанг, поднимавшихся Д.Ригертом и другими 
выдающимися атлетами.

Таким образом, повторяю, можно быть твёрдо уверенным в 
том, что в случае с Ботевым скорость штанги измерена 
достаточно точно. И, как свидетельствуют эти измерения,
штанга опять поднялась значительно выше, чем она 
поднялась бы в случае свободного полёта после 
достижения своей максимальной скорости в финале 
посыла. То есть при толчке Ботева имеет место 
практически то же самое явление, та же самая "загадка 
дополнительного подъёма", что и при толчке Ригерта и, 
судя по всему, при всех остальных толчках.

Высота свободного подлёта штанги, согласно известным
кинематическим формулам, должна быть равна:

h = (V^2)/2g = 0,167 м

(аш равно вэ квадрат делённое на два жэ)

где h - высота подлёта, V - начальная скорость
штанги 1,81 м, g - ускорение свободного падения
(равное, как известно 9,8 м/с2).

Штанга же, как видно из графиков, поднялась от точки
6 см (где, если верить подписи под графиком 2, и
была достигнута скорость 1,81 м/с) до отметки 30 см
над нулевым уровнем - то есть штанга подлетела не на
16,7 см, а на 24 см. Таким образом, штанга Ботева
оказалась поднята на лишних 7,3 см. Это, конечно, не 
столь много, как в случае с толчком Ригерта (там 
лишними были 12 см) - но всё дело в том, что очень 
большие сомнения вызывает правильность подписей под 
графиком движения штанги Ботева. Даже на глаз видно, 
что максимум скорости лежит ниже половины деления 
в 10 см, то есть пик скорости не может быть выше 
уровня 5 см. Мои расчёты, проведённые ещё до того, как 
Оппонентом были опубликованы графики с подписями, 
показали, что максимальная скорость 1,81 м/с (правда, 
я округлял её до 1,8 м/с, поскольку не предполагал, 
что смогу, ориентируясь на один лишь график, вычислять 
параметры подъёма столь точно) была достигнута на 
уровне 3-4 см, а в точке 6 см уже произошло некоторое 
снижение скорости. Это, кстати, видно и по графику 2 - 
линия скорости после прохождения отметки 3-4 см 
смещается чуть влево на несколько пикселей. Так что 
после достижения максимальной скорости штанга 
поднялась, скорее всего, не на 24 см, а на 26-27 см, 
и, значит, дополнительная высота подлёта равна 9-10 см,
что уже достаточно близко к величине дополнительного 
подлёта в толчке Ригерта. Впрочем, даже если исходить 
из того, что максимальная скорость была всё-таки на 
уровне 6 см, то лишние 7,3 см подлёта, как отмечалось 
выше, также требуют своего объяснения.

Теперь я покадрово рассмотрю толчок Ботева, начав
делать это с первой серии кадров.

Кадр 1.  В нулевой точке атлет полностью выпрямлен и
стоит на помосте на полных ступнях. Штанга слегка
прогнута под весом дисков. Скорее всего, нулевая
скорость дисков имеет место после их лёгкого 
опускания, и сейчас в грифе запасена некоторая
потенциальная упругая энергия, ибо только этим можно 
объяснить то, что, согласно графику скорости, в 
следующий момент штанга приобретёт скорость не вниз, 
а вверх и ещё слегка поднимется над нулевой точкой - 
и лишь затем снова пойдёт вниз. Этот первый завиток 
на графике скорости представляет собой не что иное, 
как след от колебания дисков штанги, гриф которой 
лежит у атлета на плечах. Ведь не приподнимает же 
атлет штангу перед тем, как уйти вниз в разгонный 
полуприсед?

Кадр 2.  Атлет максимально ушёл вниз, при этом штанга
значительно прогнулась - гораздо сильнее, чем на
кадре 1. Это свидетельствует о том, что атлет
оказывает на гриф штанги тормозящее воздействие, а
диски по инерции продолжают опускаться.

Кадр 3.  Атлет полностью выпрямлен и находится даже
чуть выше, чем на рисунке 1 - примерно на 2 см (видно, 
что пятка Ботева чуть оторвана от помоста). Но сама 
штанга находится на том же уровне, что и на кадре 1 
(ну разве что на 0,5 см выше). То есть диски штанги 
проходят нулевую отметку. Нахождение дисков штанги на 
одном уровне, а грифа (вместе с телом атлета) - на 
разных уровнях, свидетельствует о том, что на третьем 
кадре штанга прогнута сильнее, чем на первом, то есть 
атлет ещё разгоняет штангу и давит на её гриф, а диски 
при этом из-за инерции чуть отстают (в нулевой точке 
атлет не ускорял движение штанги, и потому гриф 
прогибался только под тяжестью дисков). Вообще, на 
данной страничке кадр 3 немного смещён вверх, и это 
видно даже по изображению зрителей. Конечно, то, что я 
сейчас написал, может быть неочевидным - но я 
специально выравнивал кадры в "Фотошопе" и сравнивал 
высоты по пикселям. Судя по графику скорости, на 
уровне нулевой отметки скорость штанги близка к 
максимальной, однако впереди ещё должно произойти 
крайне небольшое повышение скорости. То есть это 
повышение произойдёт уже после полного выпрямления ног.

Кадр 4 серии 1 (или кадр 1 из серии 2).  Атлет
привстал на носки, гриф штанги находится ближе к
подбородку, атлет, судя по всему, произвёл сокращение
трапеций. Гриф штанги практически прямой - это
свидетельствует о том, что завершилось и отталкивание 
штанги от тела, то есть что высвободилась энергия 
упругой деформации грифа. По-видимому, в следующую 
долю секунды штанга отойдёт от плеч. Как я уже писал 
выше, мои вычисления и подпись под графиком 
показывают, 
что штанга находится в данном положении на 6 см выше 
нулевой отметки - при этом она уже чуть-чуть потеряла 
максимальную скорость, которая была достигнута на 
уровне 3-4 см. Таким образом, максимальная скорость 
имела место между полным выпрямлением ног (кадр 3) и 
точкой отрыва штанги (кадр 4). Соответственно, уже 
после достижения максимальной скорости сила упругости 
тела и сила трапециевидных мышц продолжали 
воздействовать на штангу, что способствовало её 
дополнительному подлёту. Таким образом, после 
достижения максимальной скорости штанги имел место не 
свободный её подлёт, а подъём с воздействием на неё 
тела атлета (именно этим обстоятельством частично и 
объясняется дополнительный подлёт штанги выше той 
точки, на которую она смогла бы подняться в свободном 
движении). Достижение штангой максимальной скорости 
уже после полного выпрямления ног свидетельствует ещё 
и о том, что даже после полного выпрямления ног атлет 
продолжал около 3-4 см разгонять штангу с силой, 
превышающей силу гравитации (увеличение скорости 
означает, что ускорение положительно). За счёт каких 
же сил атлет это делал? Во-первых, за счёт силы 
икроножных мышц - видно, что атлет поднялся на носки, 
во-вторых, за счёт силы трапециевидных мышц - видно, 
что атлет поднял плечи, и, в-третьих, за счёт сил 
упругости тела и грифа штанги, которые должны были 
начать действовать, когда ослабло разгоняющее 
воздействие ног.

По всей видимости, после кадра 3 начинается торможение 
тела атлета, первое время двигавшегося вместе со 
штангой. Это торможение тела атлета, а также то 
небольшое увеличение скорости самой штанги, которое 
происходит уже после выпрямления ног, придают телу и 
штанге начальную скорость разлёта, в результате чего и 
происходит отход штанги от тела (а до того имело место 
лишь отделение тела атлета с плотно лежащей на его 
плечах штангой от помоста). При этом по графику 
скорости видно, что силы, действовавшие после 
выпрямления ног атлета, превышали силу тяжести лишь в 
течение очень короткого периода, поскольку, пройдя 
3-4 см после выпрямления ног и достигнув максимальной 
скорости, штанга затем начала её снижать. Связано это 
снижение скорости с тем, что все участвовавшие в 
подъёме упругие элементы быстро отдали запасённую 
энергию и уменьшили своё воздействие; силы же 
сокращавшихся икроножных и трапециевидных мышц, в свою 
очередь, уменьшились потому, что эти мышцы прошли 
точку оптимума развития силы. Скорость штанги 
относительно тела возросла - хотя мышцы вследствие 
большей быстроты сокращения стали развивать меньшую 
силу. Силы упругости, а также силы трапециевидных и 
икроножных мышц действовали на штангу через плечи и в 
дальнейшем до достижения высоты 6-7 см, на которой 
произошёл отрыв её, штанги, от плеч. Но величина этих 
сил стала уже меньше величины силы тяжести, поэтому 
штанга начала терять скорость. Однако терялась данная 
скорость всё же не так быстро, как она терялась бы без 
действия вышеуказанных сил.

На графике скорости видно ещё и то, что вклад 
вышеописанных дополнительных сил в конечную
максимальную скорость разгона штанги относительно
помоста не столь велик, как вклад ног, осуществлявших 
разгон. Эти силы добавляют не более 0,02-0,03 м/с к 
скорости, приданной штанге во время разгона ногами. Но 
зато эти силы поднимают штангу на дополнительные 
3-4 см, прежде чем скорость штанги начинает падать 
(без этих сил скорость не только не прирастала бы, но, 
наоборот, сразу бы уменьшилась, едва только ноги 
закончили свою работу). Эти силы играют важную роль и 
в последующей более медленной потере штангой скорости, 
поскольку продолжают противодействовать силе 
гравитации (как это видно из кадра 4 первой серии и 
кадра 1 второй серии, данные силы могут действовать 
вплоть до поднятия штанги, как минимум, на высоту 
6-7 см над нулевой отметкой). И, конечно, эти силы 
играют основную роль в остановке тела, поднимающегося 
вместе со штангой при разгоне вверх - то есть именно 
эти силы сообщают практически всю величину скорости 
отлёта штанги от тела (напоминаю: в результате 
действия этих сил происходит остановка тела и 
небольшое ускорение штанги - в результате чего телу и 
придаётся итоговая скорость отлёта от штанги).

Теперь, надеюсь, понятно, почему Составитель счёл,
что именно указанные силы способствуют уходу атлета
под штангу. Мало того что эти силы сообщают штанге
скорость относительно тела, но они ещё увеличивают
высоту подлёта штанги, поскольку максимальная
скорость штанги (по которой рассчитывается 
теоретическая высота подлёта) была достигнута ещё 
до отделения штанги от плеч атлета.

На этом я закончу разбор первой серии кадров. Тем
более, что в моём распоряжении есть более подробная
раскадровка (серия 2), да ещё с точным определением
скоростей и высот.

Но для начала я вернусь к высотам подлёта штанги и
сделаю кое-какие предварительные выкладки. Чтобы не
заниматься лишними расчётами по кадрам, я отбрасываю
свои сомнения по поводу подписей под графиком 2,
гласящих, что штанга имела максимальную скорость
1,81 см именно на высоте 6 см. При данной скорости
штанга поднялась на высоту 24 см. С каким же средним
ускорением она двигалась?

Для ответа на этот вопрос я воспользуюсь формулами
равноускоренного движения.

(В приводимых ниже уравнениях я применил следующие 
обозначения:

h - пройденный штангой путь 

Vо - начальная скорость 

Vк - конечная скорость, достигнутая по прошествии пути 
h 

a - ускорение, с которым движется штанга 

t - время преодоления пути h 

^2 - знак возведения в квадрат 

x - знак умножения)


h = Vоt + a(t^2)/2                         (1)

Vк = Vо - at                               (2)

Из уравнения (2) я нахожу, что

t = (Vо - Vк)/a.

Подставив данное значение t в первое уравнение, я 
получаю:

h = Vо(Vо - Vк)/a + a(((Vо - Vк)/a)^2)/2

Откуда следует, что

a = Vо(Vо - Vк)/h + ((Vо - Vк)^2)/2h       (3)

По скорости в начале и конце интервала движения
штанги и пройденному штангой расстоянию можно найти
ускорение, с которым она двигалась. Кроме того, зная
ускорение штанги, можно найти и действовашие на неё
силы. Из второго закона Ньютона (произведение массы
штанги "m" на её ускорение "a" равняется сумме сил,
действующих на штангу - а на штангу в данном случае
действует сила атлета F (через его плечи и руки), а 
также сила тяжести mg (g - это ускорение свободного 
падения, равное минус 9,8 м/с2)) следует, что

ma = F + mg

Отсюда:

F = ma - mg = m(a - g)                     (4)

То есть, зная ускорение движения штанги и её вес,
можно найти ту силу, с которой атлет действовал на
штангу во время толчка (при подстановке величин в 
формулы следует учитывать знаки скорости и ускорений, 
а то я записал формулы в общем виде, без учёта 
направления движения и сил).

Итак, в рассматриваемом случае Vо = 1,81 м/c, Vк = 0
(поскольку штанга останавливается в верхней точке),
h = 0,24 м, g = - 9,8 м/c2, m = 250 кг.

В результате подстановки этих данных в уравнение (3)
получается, что

a = - 6,8 м/с2

То есть штанга теряла скорость в среднем с ускорением, 
на 3 м/с2 меньшим, чем это было бы в случае её
свободного движения.

Какова же была сила атлета, мешавшая штанге терять 
скорость подъёма?

Я подставляю найденное значение ускорения в формулу
(4) и получаю F = 750 Н

Итак, уже после достижения штангой максимальной
скорости атлет воздействовал на штангу от точки
достижения максимальной скорости до точки
максимальной высоты подъёма со средней силой 750 Н.
Велика ли эта сила? Эта сила развивается при
удержании или очень медленном неускоренном жиме 
76,5 кг. Думаю, что такой атлет, как Ботев, может 
чисто, без так называемых "фуксов" выжать не менее 
150 кг, то есть развить в жиме силу порядка 1470 Н. 
В два раза меньшую силу Ботев развил при толчке от 
груди потому, что жимовое движение во время толчка он
выполнял с очень большой скоростью, а сила мышц при
быстром сокращении значительно снижается. Но, может
быть, на той скорости, с которой Ботев вынужден при 
толчке от груди сокращать мышцы рук, просто нельзя 
развить силу в 750 Н? Тогда загадка дополнительной 
высоты подъёма станет просто неразрешимой. Как же 
оценить величину силы, развиваемой при высокой 
скорости сокращения мышц? Способ такой оценки был 
предложен ещё в тексте "О быстроте разгибания рук при 
толчке от груди". Согласно этому способу оценки, 
даже самые сильные штангисты за четверть секунды не 
могут выжать от груди стоя штангу весом более 40 кг. 
Можно вычислить, какова должна быть сила атлета, чтобы
он стоя мог выжать за четверть секунды хотя бы 30 кг.

В начале жима скорость штанги равна нулю, и в своём
стремлении выжать штангу максимально быстро атлет
должен стремиться прикладывать к ней максимальную
силу и двигать штангу с максимально большим 
ускорением. Конечно, при реальном подъёме сила 
воздействия атлета на штангу постоянно меняется, а 
значит, меняется и то ускорение, с которым она 
движется. Однако можно найти просто среднее ускорение, 
с которым нужно двигать штангу, чтобы поднять её за 
четверть секунды на высоту выпрямленных рук (у Ботева 
эта высота равна примерно 0,55 м).

Искомое ускорение составляет

a = 2h/(t^2) = 17,6 м/с2

где h - высота подъёма, а t - время подъёма.

Таким образом, чтобы выжать штангу за 0,25 с, нужно 
поднимать её с ускорением 17,6 м/с2.

Используя второй закон Ньютона, можно найти, какую
среднюю силу нужно приложить к штанге весом 30 кг,
чтобы поднимать её с ускорением 17,6 м/с.

ma = F + mg

F = ma - mg = m(a - g) = 30 x (17,6 - (- 9,8)) = 
= 30 x 27,4 = 822 Н

Итак, для того, чтобы выжать 30 кг за четверть
секунды, нужно прикладывать к штанге среднюю силу
примерно в 822 Н, что соответствует примерно 84 кг.
Может ли такой атлет, как Ботев, выжать 30 кг за
четверть секунды? Думаю, это ему вполне по силам. А
значит, ему вполне по силам в жиме со средней
скоростью штанги относительно тела

V(ср) = h/t = 0,55/0,25 = 2,2 м/c

развить среднюю силу в 822 Н. При толчке штанга
относительно тела проходит тот же путь 0,55 м (равный,
напоминаю, длине выпрямленной кверху руки)
приблизительно за то же самое время - 0,25 с, а значит,
мышцы сокращаются с той же средней скоростью, что и 
при скоростном жиме, а значит, для мышц Ботева не 
будет проблемой развить среднюю силу 750 Н и во время 
толчка.

Однако жим штанги осуществляется с опорой ногами о
помост, а при толчке от груди атлет какое-то время
находится в безопорном состоянии. Если присмотреться
к кадрам толчка Ботева, то оказывается, что в
безопорном положении Ботев находился далеко не всё
время после отрыва штанги от плеч - это имело место
только на двух кадрах из четырнадцати (на четвёртом
и пятом кадрах).

Указанные под графиками серии 2 высоты подлёта
штанги и соответствующие этим высотам величины
скорости позволяют воспользоваться формулами (3) и
(4) и вычислить уже не среднее ускорение и среднюю
силу на всём протяжении движения, а среднее
ускорение и среднюю силу на отдельных участках
траектории штанги - это позволяет нарисовать более 
точную и полную картину толчка. Все свои расчёты я 
здесь приводить не буду - я буду приводить лишь их 
итоги. И ещё раз напоминаю, что не следует забывать 
про смещение графиков и кадров.

Итак, от положения штанги на 2 см ниже нулевого
уровня до кадра 1 штанга движется с ускорением 
0,89 м/с2, а атлет действует на штангу с силой в 
2674 Н. Эта сила превышает силу, необходимую для 
неподвижного удержания 250 кг, что и понятно: 
атлет на этом участке ещё подталкивает штангу 
кверху силой бёдер, затем силой икр, трапеций и, 
наконец, силой упругости позвоночника и плечевого 
пояса, разгоняя снаряд до максимальной скорости.

Между кадрами 1 и 2 штанга движется с ускорением
минус 4,89 м/с2, то есть штанга уже теряет скорость.
Но теряет она её медленнее, чем при совершенно
свободном полёте, так как атлет действует на неё с
силой 1229 Н. Эта сила эквивалентна силе удержания
или медленного неускоренного жима 125 кг. И, судя по
кадру, это действительно жим: атлет упирается
носками в пол и совершает посыл штанги от положения
касания штангой плеч (кадр 1) до той точки, в
которой штанга уже достаточно удалилась от плеч
(кадр 2).

От кадра 2 до кадра 3 штанга тормозится уже с
заметно большим ускорением - минус 7,69 м/с2. А
значит, атлет воздействует на неё уже слабее - со
средней силой всего 527 Н (что эквивалентно
удержанию или неускоренному жиму 54 кг). Сила резко
падает потому, что на предыдущем участке ещё
действовал посыл плечами, который и дал основную
силу воздействия, а на этом участке происходит уже
чистый жим штанги.

Между кадрами 3 и 4 штанга движется с ускорением
минус 6,56 м/с2, а сила, развиваемая атлетом, равна
810 Н (что эквивалентно удержанию или неускоренному
медленному жиму 82,5 кг). Кого-то может удивить
столь резкое возрастание силы на этом участке,
однако ничего удивительного тут нет. Дело в том, что
чем выше поднимается штанга, тем медленнее она
движется. На кадре 3 (в начале рассматриваемой фазы)
скорость штанги равна 1,35 м/с, а в начале
предыдущей фазы она была равна 1,65 м/с. Атлет же,
как видно из кадров, ещё и не думает набирать
скорость вниз, то есть скорость штанги падает как
относительно помоста, так и относительно атлета, а
значит, падает и скорость сокращения мышц, и,
следовательно, возрастает сила их сокращения. Отсюда и
прибавка в итоговой силе воздействия на штангу.

Между кадрами 4 и 5 атлет и штанга находятся в
свободном полёте. Однако, несмотря на это, штанга
тормозится не с ускорением свободного падения, а с
ускорением минус 7,17 м/с2, то есть атлет
по-прежнему воздействует на штангу, и сила его
воздействия равна 658 Н. Это ещё раз показывает, с 
какой силой атлет может отталкивать штангу от тела
(или тело от штанги), находясь в свободном полёте.

Между кадрами 5 и 6 атлет и штанга всё ещё находятся
в свободном полёте, но атлет уже ушёл вниз - то есть 
скорость атлета относительно штанги возросла, а 
значит, сила воздействия на штангу на этом участке
должна быть ниже, чем на интервале 4-5. Однако
вычисления показывают нечто на первый взгляд 
достаточно странное: штанга Ботева двигалась на этом 
участке с ускорением минус 10,8 м/с2, то есть штанга
тормозилась сильнее, чем её могла затормозить одна
лишь гравитация. Надо признаться, поначалу эти
данные привели меня в замешательство. Торможение
на 1 м/с2 большее, чем при свободном падении, должно
было свидетельствовать или об ошибках аппаратуры,
или о гравитационной аномалии на Олимпиаде в
Атланте. Ну действительно: разве мог атлет не только
не ускорять штангу, но ещё и тянуть её вниз с силой
в 256 Н?

Хорошенько поразмыслив, я пришёл к выводу, что
причиной данной "гравитационной аномалии" могут быть
упругие колебания частично освободившегося от
нагрузки грифа. После того как атлет оторвался от
помоста между кадрами 3 и 4, давление плеч на гриф
ослабло, и гриф стал выпрямляться, избавляясь от
деформации. Это избавление грифа от упругой деформации 
почти никак не проявлось на этапе 4-5, но вот на этапе 
5-6 уже существенно ослабило воздействие рук на диски 
штанги. Дело в том, что скорость ухода атлета вниз 
постепенно возрастала - следовательно, возрастала и 
скорость сокращения мышц. Соответственно, сила 
сокращения мышц упала, а значит, упало и воздействие 
рук на гриф штанги (возможно, даже до нуля). Поэтому 
где-то между кадрами 4 и 5 гриф полностью выпрямился и 
начал не ускорять, а, наоборот, тормозить пытающиеся 
обогнать его диски. Вот это быстрое торможение дисков 
грифом и дало дополнительного отрицательное ускорение 
в 1 м/с2. Судя по надписи под графиками, данное 
дополнительное торможение произошло на участке подъёма 
длиной всего 2 см. При средней скорости штанги 
0,58 м/с дополнительное торможение длилось лишь 
0,034 с, и скорость штанги за счёт этого упала всего 
лишь на 0,034 м/с (за тот же самый период уменьшение 
скорости штанги в результате действия гравитации 
составило 0,33 м/с, то есть оказалось в десять раз 
большим).     

На участке между 6-м и 7-м кадрами ноги атлета уже
стоят в "ножницах", однако он продолжает опускать
тело и одновременно воздействовать на штангу,
которая ещё движется по инерции вверх. Расчёты
показывают, что штанга движется на этом участке с
ускорением минус 8 м/с, то есть атлет воздействует
на неё с силой всего 450 Н, что эквивалентно
медленному жиму 46 кг. На кадре 7 достигнута
максимальная высота подъёма дисков. На этом кадре 
атлет уже твёрдо стоит ногами на помосте, а значит, 
силы, возникающие при расстановке ног и тянущие тело 
атлета вниз (о них упомянуто в текстах "О быстроте 
разгибания рук при толчке от груди" и "На Алборова не 
обижаются"), уже не действуют. Похоже, они действовали 
чуть раньше, и большей частью именно за счёт этих сил 
было осуществлено быстрое опускание тела на двух
предыдущих этапах свободного полёта - так что в
какой-то момент выжимающее воздействие рук на штангу 
было даже равно нулю. Руки просто быстро выпрямлялись, 
не прикладывая к грифу никакого подъёмного усилия 
(именно этим обстоятельством объясняется резкое 
падение силы воздействия на штангу на этапе 5-6). 
Дальнейшее опускание тела на этапе 6-7 происходило уже 
за счёт силы тяжести и инерции и, конечно, 
отталкивания от штанги в стремлении полностью 
выпрямить руки.

Между кадрами 7 и 8 штанга начинает опускаться 
(на 1 см), а атлет, немного оседая, начинает
принимать на себя вес штанги. Ускорение штанги равно
минус 5,12 м/с, а воздействие атлета на штангу равно
уже 1170 Н.

Между кадрами 8 и 9 штанга заметно прогнулась. То
есть атлет тормозит гриф штанги, а диски прогибают
гриф собственной тяжестью и инерцией, поскольку идут 
уже вниз). При этом диски движутся с ускорением минус 
3,69 м/с2, а сила воздействия на диски равна 1528 Н.

Между кадрами 9 и 10 атлет остаётся практически в
той же позиции, а диски опускаются ещё ниже и гриф
прогибается ещё сильнее. Ускорение дисков становится
равным плюс 0,49 м/с2, то есть диски теряют
набранную вниз скорость, а сила атлета, которая на
них воздействует через гриф, равна уже 2573 Н. То
есть это сила неускоренного подъёма или удержания
263 кг. Понятно, что такую силу атлет развивает уже
не за счёт силы мышц рук, а за счёт усилий ног,
опирающихся о помост в "ножницах". Именно из-за того, 
что эта сила эквивалентна силе удержания штанги массой 
263 кг, штанга весом в 250 кг снижает скорость своего 
движения вниз. (Кстати, если бы сила воздействия была 
в точности равна силе удержания массы 250 кг (2450 Н), 
штанга продолжала бы опускаться с прежней скоростью, 
а если бы сила оказалась меньше 2450 Н, то штанга, 
опускаясь, увеличивала бы скорость движения.)

Между кадрами 10 и 11 атлет стоит фактически в той же 
позе, штанга по-прежнему движется вниз, но ускорение, 
с которым она тормозится атлетом, становится равным 
уже 3,36 м/с2, а сила воздействия атлета на штангу 
возрастает до 3290 Н (то есть становится эквивалентной 
силе удержания 336 кг.

Между 11-м и 12-м кадрами штанга смещается вниз ещё на
1 см, между тем как абсолютная величина её
ускорения, направленного вверх (которое пока так и
не остановило штангу) возрастает до 4,08 м/с2. При
этом сила атлета, воздействующая на штангу,
становится равной 3470 Н, что эквивалентно 354 кг.
То есть атлет, тормозящий в "ножницах" падающую на
него штангу весом 250 кг, испытывает перегрузки, как
при удержании на прямых руках штанги весом в 354 кг.

Между 12-м и 13-м кадрами, согласно подписям под
графиками (график 14 соответсвует кадру 13), со
штангой опять происходить нечто странное. На кадре 12 
(график 13) штанга движется вниз со скоростью 
0,12 м/с2, а на кадре 13 (график 14) штанга движется
по-прежнему вниз, но с гораздо меньшей скоростью
(0,02 м/с2). То есть штанга вроде бы всё время
двигалась вниз, но тут почему-то, согласно подписям
под графикам, поднялась на 1 см. Как такое может
быть? Может, что-то напутали исследователи? Нет,
дело тут, видимо, в следующем: на этом участке
штанга уже успела остановиться, пойти вверх и
подняться на 1 см или даже более, а затем опять
остановиться и пойти вниз - то есть снова приобрести
отрицательную скорость. Таким образом, при остановке
штанги в нижней точке диски не замирают, а совершают
упругие колебания - их-то, по-видимому, и фиксирует
прибор. А потому считать среднее ускорение на
участке между 12-м и 13-м кадрами не имеет смысла -
ведь ускорение дисков на этом участке было то
положительным, то отрицательным, и оценить по такому
среднему ускорению силу воздействия никак нельзя.
Впрочем, для понимания сути происходящего это уже не
очень-то и нужно. Собственно, картина подъёма и так
достаточно ясна, загадка дополнительного подъёма
штанги уже раскрыта.

Решение этой загадки состоит в следующем: штанга 
поднимается на лишние 7-10 см потому, что после 
окончания её разгона из полуприседа атлет воздействует 
на гриф силой жимовых мышц. То есть атлет совершает 
скоростной жим штанги, развивая на разных участках 
толчка силу от 1229 Н (в самом начале движения) до 
нуля (в середине движения во время собственно ухода 
тела вниз) и около 450 Н на завершающем этапе - так 
что средняя сила воздействия жимовых мышц на штангу 
составляет 750 Н.

Воздействие рук (и, конечно, всех других жимовых
элементов) на штангу во время толчка никак нельзя 
назвать незначительным, нет - оно оказывает очень 
заметное влияние на результаты толчка. Без этого 
воздействия толчок просто не состоялся бы, так как 
тут и штанга поднялась бы на недостаточную высоту, и 
тело атлета не столь быстро ушло бы вниз под штангу.

Таким образом, Роман и Шакирзянов были, конечно,
неправы, когда писали о каких-то огромных ускорениях 
штанги в результате отталкивания атлетом ногами от 
помоста уже после отрыва штанги от плеч. Воздействие 
на штангу было хотя и переменным, но длилось 
практически в течение всего времени распрямления рук, 
и связано оно именно с воздействием на штангу жимовых 
мышц. Учитывать движение ног в данном случае нужно 
только в плане наличия или отсутствия опоры тела 
атлета при жиме штанги, так что поведение ног с 
результатами воздействия на штангу связано именно в 
этом ключе. Составитель, критикуя объяснение Романа и
Шакирзянова, в целом правильно описал силы,
действующие на штангу при толчке, но недооценил
возможную силу воздействия жимовых мышц на штангу, в
результате чего и был вынужден искать решение
загадки дополнительного подлёта в занижении Романом
и Шакирзяновым измеренной скорости движения штанги.
Критиковавший же Составителя Алборов просто не понял
его текст и не совсем точно оценил все силы,
действующие при толчке, а также последовательность
их включения (и в результате не увидел, что атлет
может воздействовать на штангу плечами даже после
достижения максимальной скорости разгона штанги). Но
зато Алборов верно указал, что дополнительная высота
подлёта штанги связана с воздействием атлета на
штангу руками в течение всего времени толчка. Алборову 
следовало бы только подтвердить это цифрами, но этого
он, к сожалению, не сделал.

Решение загадки дополнительного подъёма наталкивает
на следующий вывод: поскольку сила рук имеет большое
значение для успеха подъёма околопредельных весов,
её необходимо полноценно тренировать. Но только
здесь важна не сила рук вообще, а именно та сила,
которую руки развивают на высокой скорости, то есть
здесь важна сила рук при таком их распрямлении, 
которое длится приблизительно 0,25 с. Для атлетов 
экстра-класса это веса в 25-30 кг, а для атлетов 
уровня пониже - ещё меньше. Для штангиста важно уметь 
как можно быстрее выжать этот лёгкий вес, то есть 
приложить к нему как можно большую силу и тем самым 
придать штанге как можно большее ускорение. Ибо во 
время толчка руки (а также плечи и иные жимовые 
элементы) атлета работают именно в таком режиме.

Разумеется, мои слова не нужно воспринимать как
призыв обязательно тренировать мышцы скоростными
жимами: я не уверен, что это чем-то лучше, нежели
простая тренировка толчка - ведь при толчке мышцы
работают как раз в самом нужном режиме. Скоростные
же жимы лёгкой штанги мало способствуют
дополнительной гипертрофии мышц. Гипертрофию мышц
вызывают именно тяжёлые жимы. И связана данная
гипертрофия главным образом с фазой опускания, а
вовсе не подъёма веса. При подъёме лёгкой штанги к
ней можно приложить такую же силу, как и при подъёме
более тяжёлой штанги (если разгонять лёгкую штангу с
большим ускорением), но при подъёме лёгкой штанги
нельзя развить большую силу при опускании веса - по
этой-то причине гипертрофия мышц и наблюдается
только после тяжёлых жимов. Однако жим больших весов
связан с приспособлением мышц именно к опусканию
большого веса - что, с одной стороны, способствует
гипертрофии мышц, но, с другой стороны, сопряжено с
замедлением сокращения мышц, с приобретением
волокнами, включёнными в опускание веса, "медленных" 
свойств ("медленные" волокна - это антигравитационные 
мышцы, которые приспособлены как раз для опускания 
веса; "быстрые" же волокна не приспособлены для 
опускания веса, их задача - скоростные позитивные 
движения). Именно в этом и состоит объяснение так 
называемого "закрепощения мышц" при их "накачке" 
(это я уже выдаю результаты моих самых последних 
теоретических исследований). Известно, например, что 
атрофия мышц после космических полётов, вызванная 
отсутствием в условиях невесомости негативной фазы 
движения, связана с атрофией главным образом 
"медленных" волокон и приобретением ими свойств 
"быстрых" волокон. Что в итоге приводит к увеличению 
скорости сокращения мышц в целом. Но вместе с тем 
данная атрофия "медленных" волокон вследствие их 
неиспользования приводит к падению общей силы мышц. 
Для штангиста же важен баланс между гипертрофией мышц 
и скоростью их сокращения. Как достичь этого баланса, 
я, увы, пока толком не знаю.

Из проделанного анализа следует сделать ещё один
вывод. Равенство максимальных скоростей при посыле
во время толчка и равенство необходимых высот
подъёма штанги ещё не дают возможности взять штангу
на грудь в "ножницы", по глубине существенно не
отличающиеся от "ножниц" при толчке от груди. Сие 
будет возможно только в том случае, если после набора
максимальной скорости атлет во время подрыва штанги
имеет точно такую же возможность воздействия на
штангу, как и при толчке от груди. А в этом у меня 
есть глубокие сомнения. Жим штанги не идёт ни в какое
сравнение с тягой штанги к груди. Поднимая штангу на
полусогнутых руках (как это происходит после подрыва
во время взятия на грудь) от уровня лучевого
сплетения до высоты фиксации штанги на груди руки
способны развить минимальную силу, ни в какое
сравнение не идущую с силой рук при жиме в верхней
амплитуде движения.

Но сейчас я ничего утверждать не стану, это пока
только моё поверхностное мнение. Если имеется точно
такая же раскадровка с цифрами взятия на грудь (как
в случае с толчком), то я могу провести анализ и
посчитать силу воздействия рук на штангу в этом
случае, и сравнить её с силой воздействия рук на
штангу при толчке. Это и даст ответ на вопрос: можно
ли взять на грудь в "ножницы" тот же вес, что и
толкнуть от груди в "ножницы", кардинально не
увеличивая скорость разгона штанги при подрыве (то,
что потребуется некоторое её увеличение, которое
может быть достигнуто направленной тренировкой,
признаёт и сам Составитель).

Дабы у участников дискуссии не сложилось
неправильное представление о некоторых моментах, я
хочу ещё раз обратить внимание на тот факт, что
Составитель принципиально правильно описывал
ситуацию с толчком в тексте "О быстроте разгибания
рук при толчке", которую непонятно зачем критиковал 
Алборов. Вот слова Составителя:

"При посыле, в момент полного выпрямления атлетом
ног, когда штанга ещё не успела отделиться от плеч
атлета, то есть ещё продолжает лежать на его
дельтовидных мышцах, она обычно достигает своей
максимальной вертикальной скорости. Но, как
обнаружили скрупулёзные исследования, этой пусть
даже и максимальной скорости оказывается совершенно
недостаточно для того, чтобы штанга поднялась на
необходимую для её подхвата и дальнейшей фиксации
высоту. Поэтому если не включатся упоминавшиеся выше
механизмы, то свободно отпущенный снаряд подлетит по
инерции лишь примерно на половину нужной для фиксации 
высоты. И именно отталкивание грифа плечами - как за 
счёт силы трапециевидных мышц, так и за счёт упругой 
реакции всего плечевого пояса и позвоночного столба - 
придаёт снаряду необходимые для его дополнительного 
подлёта ускорение, скорость, энергию (напоминаю, что, 
по моим расчётам, на этапе, включающем контакт штанги 
с плечами и отлёт на небольшое расстояние от плеч, 
атлет воздействовал на штангу со средней силой 
1229 Н - В.П.). Самую же последнюю небольшую порцию 
энергии, поднимаемому до момента окончания подседа 
снаряду передают, естественно, сами руки - силой 
своих разгибателей (как показывают расчёты, сила 
воздействия рук в зависимости от этапа толчка менялась 
от 800 Н до нуля - В.П.)."

Однако уже во второй своей статье-ответе на критику
Алборова Составитель решил, что описанных им сил
недостаточно для подъёма штанги, и предположил что
Роман и Шакирзянов занизили скорость штанги, измеряя
её по центру грифу. Впрочем, как мерили скорость в
случае толчка Ригерта, я не знаю, а потому ничего
утверждать не могу, - может, скорость там и впрямь
была занижена. Всё-таки разница в скорости между
толчком Ригерта (1,56 м/с) и толчком Ботева 
(1,81 м/с) существенна. При этом, несмотря на то что
скорость у Ригерта была меньше, чем у Ботева, штанга
у Ригерта, наоборот, поднялась на относительно
большую высоту (дополнительные 12 см против 7-10 см
Ботева) - хотя, возможно, это объясняется более
резвыми руками Ригерта и более лёгкой штангой в его
случае. Так что для толчка Ригерта нужно провести
аналогичный анализ, чтобы досконально с ним
разобраться.

Самое интересное, что Алборов в своей критической
статье написал практически то же самое, что и
Составитель:

"На основании изложенного можно предположить, что:

1. Главную роль в придании начальной скорости
снаряду при толчке от груди играют разгибатели ног.

2. Дополнительное ускорение при толчке от груди
штанга получает за счёт упругости плечевого пояса
спортсмена и упругой деформации грифа штанги.
Воздействие этих сил на снаряд прекращается в момент
отделения штанги от груди.

3. В обеспечении необходимой высоты подлёта штанги
участвуют и разгибатели рук, препятствующие
замедлению движущейся вверх штанги. При этом
вертикальное воздействие рук на гриф штанги должно
быть постоянным с момента отделения штанги от груди
до полной её фиксации на выпрямленных руках.

4. При финальном разгоне штанги в толчке от груди
очень важно полностью вытянуться вверх с выходом на
носки. Это даст возможность для лучшего разгона
штанги и увеличит высоту подлёта снаряда.

5. Правильное, рациональное исполнение перечисленных
факторов, наверное, обеспечит подлёт штанги на
требуемую высоту."

С этими словами Алборова трудно не согласиться, но и
Составитель в первой статье говорил об этом же.
Зачем Алборов затеял этот спор - вообще непонятно.
Похоже, это просто недопонимание (или личные
мотивы?). Хотя, судя по всему, причина была в том,
что Алборов всё-таки не совсем точно представлял
себе все нюансы толчка и не учитывал, что
максимальная скорость достигается ещё при контакте
грифа с плечами атлета, а потому пытался критиковать
Составителя за то, что тот включил данные силы в
разряд сил, способствующих дополнительному подлёту
штанги уже после разгона её до максимальной
скорости.
 
Ответить

Тема: Re: О силе воздействия на штангу при толчке от груди
Автор: Чумбурмучка
Дата: 16/03/2004 21:24
 	
В конце концов все эти расчёты не отменяют очевидного 
факта - существенную потерю штангой набранной высоты 
при подъёме на грудь в "разножку". Даже в учебниках по 
тяжёлой атлетике есть рисунки, показывающие траекторию 
центра тяжести штанги при подъёме штанги на грудь 
способами "разножка" и глубокие "ножницы". Из этих 
рисунков ясно видно, что потеря набранной высоты при 
"разножке" на 20-30% больше, чем при "ножницах". 
Причём там же отмечается, что вставать из "разножки" 
(низкого седа) тяжелее, чем из низких "ножниц". 

Я думаю, что если использовать средние "ножницы" 
вместо низких, то потери высоты могут быть ещё меньше, 
а вставание из подседа ещё более лёгким.
 
Ответить	Открыть подтему

Тема: Эх, опубликую побыстрее, пока не опередили...
Автор: Составитель
Дата: 29/03/2004 10:54
 	
В последнее время мне никак не удаётся нормально
посидеть и написать обстоятельные ответы Вадиму, 
Мовлади, СиРоТКе и Каа (не распространяясь уж о том 
товарище, который давно ругает меня на форуме 
Мовлади). Основная часть планируемых ответов будет 
носить совершенно рутинный характер. Но вот в мою 
рецензию на исследование Вадимом подъёма от груди 
будет включена одна идея, додумываясь до которой, я 
вынужден был немножко поломать себе голову (голову я 
поломал себе действительно совсем "немножко", видимо, 
лишь потому, что направление этого ломания было 
изначально подсказано мне тем подходом, который 
применил в своём исследовании Вадим). Опасаясь, что 
кто-нибудь опередит меня в изложении указанной идеи, я 
и решил как можно быстрее изложить её здесь, на форуме.

В своей работе Вадим написал следующее:

"Итак, уже после достижения штангой максимальной
скорости атлет воздействовал на штангу от точки
достижения максимальной скорости до точки
максимальной высоты подъёма со средней силой 750 Н.
Велика ли эта сила? Эта сила развивается при
удержании или очень медленном неускоренном жиме
76,5 кг. Думаю, что такой атлет, как Ботев, может
чисто, без так называемых "фуксов" выжать не менее
150 кг, то есть развить в жиме силу порядка 1470 Н.
В два раза меньшую силу Ботев развил при толчке от
груди потому, что жимовое движение во время толчка он
выполнял с очень большой скоростью, а сила мышц при
быстром сокращении значительно снижается." 

Предположение Вадима насчёт того, что Ботев "может
чисто, без так называемых "фуксов" выжать не менее
150 кг", скорее всего, соответствует действительности 
- насколько я знаю, чемпион мира в категории 100 кг 
Павел Кузнецов, толкавший на тренировках 247,5 кг, 
чисто выжимал стоя как раз 150 кг. Однако у меня 
вызывает большое сомнение само вот это соотношение 
величин 150 кг и 76,5 кг. Неужели при переходе от 
молниеносного движения к медленному величина 
прикладываемой силы возрастает всего лишь вдвое? Как 
мне кажется, сила должна увеличиться тут, как минимум, 
втрое. 

Примерно такое трёхкратное увеличение силы на самом 
деле тут и происходит. Оценивая силу рук при медленном 
жиме стоя штанги весом 150 кг, Вадим исходил из 
неточных представлений - из того, что при медленном 
жиме руки развивают силу, идеально равную весу штанги. 
А в реальности на большей части траектории подъёма 
медленным жимом руки развивают (и, тем более, могут 
развить) силу, существенно большую, чем 150 кг. Силу, 
равную 150 кг, руки развивают только в "мёртвой" точке 
медленного жима. На остальных же участках медленного 
жима сила у рук, повторяю, заметно выше. И она, сила, 
на этих участках может ещё даже возрасти, если снизить 
скорость подъёма.

То есть при вынесении оценки силе рук Ботева во время 
медленного жима Вадим исходил примерно из таких же 
неверных представлений, из каких когда-то исходил я 
сам, когда писал, что сила рук при толчке от груди в 
точности равна весу штанги, выжимаемой за четверть 
секунды. Мне в моём случае надо было к весу штанги 
прибавить ещё и ту силу, которая обеспечивает высокую 
скорость подъёма данной штанги, а Вадиму в его случае 
нужно было учесть величину не минимальной силы в 150 
кг, а все величины той силы (точнее, её среднюю 
величину), которую атлет прикладывает (или способен 
приложить) во время жима стоя 150 кг.

Ещё раз: в процитированном абзаце Вадим "подстриг под 
одну гребёнку" два достаточно разных проявления силы 
рук: среднюю силу рук при толчке от груди и минимальную
силу рук в "мёртвой" точке при медленном жиме стоя. А 
надо было в обоих случаях выяснять и сравнивать именно 
средние силы рук.

При толчке от груди реальная сила рук, прикладываемая 
Ботевым к грифу, меняется, допустим, таким образом: в 
самом начале движения она равна ста двадцати 
килограммам, к нижней четверти пути движения падает до 
нуля, от нижней четверти до верхней четверти пути 
движения (то есть на половине пути) продолжает 
оставаться равной нулю и затем на завершающей четверти 
быстро возрастает от нуля до трёхсот пятидесяти 
килограммов. И среднее усилие составляет здесь, таким 
образом, семьдесят шесть с половиной килограммов. В 
свою очередь, при медленном жиме стоя в начале 
движения руки прикладывают к грифу силу порядка 
двухсот килограммов, к середине движения эта сила 
уменьшается до ста пятидесяти пяти килограммов и затем 
опять нарастает до двухсот килограммов. Среднее усилие 
получается тут равным ста семидесяти пяти килограммам. 

Однако даже такое распределение усилий при реальном 
медленном жиме всё равно не отражает истинной силы 
рук. При статичных усилиях в нижней части траектории 
медленного жима руки способны на показатели порядка 
двухсот двадцати килограммов, в средней части 
траектории подъёма - на статичные усилия порядка ста 
шестидесяти килограммов (а возможно, и чуть больше - 
ведь в процессе жима снизу дошедшие до середины пути 
руки оказываются уже слегка утомлёнными, поэтому при 
статичном жиме в свежем состоянии они в "мёртвой" 
точке должны показать несколько большую силу), а в 
верхних частях траектории подъёма руки могут показать 
статическую силу порядка двухсот пятидесяти - 
четырёхсот килограммов. (Для тех, кто не понял, какое 
отношение измерения статической силы имеют к подъёму 
реального веса: данные измерения дают достаточно 
правильные представления о медленном жиме штанги с 
меняющимся описанным образом весом - конечно, в 
реальности такой штанги с меняющимся весом не 
существует, но тренажёр с подобным изменением нагрузки 
на разных высотах при большом желании сконструировать 
всё-таки можно.) И средняя сила тут будет равна 
примерно двумстам десяти - двумстам пятидесяти 
килограммам, что уже гораздо ближе к соотношению, 
заявленному в самом начале данного текста. 
   
В заключение нужно, видимо, ответить на такой вопрос: 
почему при столь высокой средней силе рук атлет может 
выжать штангу только в полтора раза меньшего веса, то 
есть почему он никак не может использовать имеющийся у 
него силовой потенциал? 

Дело в том, что почти весь данный потенциал атлета 
сосредоточен при жиме в начале и, особенно, в конце 
пути. Силовой потенциал в конце пути абсолютно 
бесполезен для преодоления "мёртвой" точки, а что 
касается силового потенциала, имеющегося в начале 
пути, то он всё же совершенно недостаточен для разгона 
штанги до такой скорости, на которой можно пролететь 
"мёртвую" точку по инерции. Силовой потенциал при жиме 
тем больше, чем ниже находится штанга. И в то же время 
он недостаточен для того, чтобы всё время увеличивать 
скорость подъёма с самого низа - ибо чем выше 
становится скорость подъёма, тем меньшее усилие на 
такой скорости мышцы оказываются способны приложить. 
Четыре эти фактора: бесполезность высокого силового 
потенциала наверху подъёма, недостаточно высокий 
потенциал мышц на нижнем участке подъёма, уменьшение 
усилия при приближении к "мёртвой" точке, а также 
падение величины усилий при высокой скорости 
сокращения мышцы в итоге и приводят к невозможности 
реализовать заманчиво высокий средний потенциал 
жимовых мышц.
 
Ответить	Открыть подтему

Тема: Re: Эх, опубликую побыстрее, пока не опередили...
Автор: VadimPro
Дата: 29/03/2004 18:52
 	
Уважаемый Составитель, я согласен далеко не со всем, 
изложенным Вами. И в первую очередь я не согласен вот 
с этими Вашими словами:

"Оценивая силу рук при медленном жиме стоя штанги 
весом 150 кг, Вадим исходил из неточных представлений -
 из того, что при медленном жиме руки развивают силу, 
идеально равную весу штанги".

Предлагая ориентироваться при медленном жиме на силу в 
150 кг, ни из каких таких представлений, я, конечно 
же, не исходил. Я прекрасно представлял, что во время 
жима сила рук постоянно меняется (потому-то я всё 
время и писал именно о средней силе). Причём в 
первоначальном варианте текста у меня было довольно 
длинное отступление (которое я в конечном итоге 
отбросил, решив излишне не загружать текст) о реальном 
распределении сил при жиме. В этом отступлении я 
писал, что хотя при скоростном жиме средняя сила 
равняется 76,5 кг, реальное распределение сил при 
жиме будет совершенно иным. Средняя сила, в 
особенности при скоростном жиме - это просто 
условность (и даже некоторая фикция), фактически не 
имеющая физического смысла. Реально в начале толчка 
штанга ещё покоится относительно тела атлета, и тут 
атлет развивает практически максимально возможную 
силу. Затем, по мере приобретения штангой скорости, 
сила воздействия на штангу может упасть практически до 
нуля, то есть сила при жиме - величина в очень 
значительной степени переменная, а вот средняя сила - 
это уже мера воздействия атлета на штангу в целом за 
весь период движения. Понимал я, конечно, и то, что 
при жиме максимального веса сила тоже меняется в 
течение всего движения, и даже если принять условие, 
что штанга в течение большей части траектории движется 
равномерно и очень медленно, то все равно, чтобы 
придать даже эту скорость штанге, вначале нужно 
приложить силу несколько большую, чем вес штанги. В 
реальности же скорость движение штанги даже при жиме 
больших весов должна меняться довольно сильно. 

Вы правы - штанга ускоряется в начале движения и 
замирает в "мёртвой" точке, чтобы затем опять слегка 
ускориться и снова замедлиться. Но мне нужно было  
сравнить среднюю силу при скоростном жиме с какими-то 
очевидными параметрами силы при максимальном усилии - 
не прибегая при этом к сложным расчётам. А с чем же 
ещё и сравнивать среднюю силу, как не со средней же 
силой? Какой смысл сравнивать среднюю силу в 
скоростном движении с максимальной силой при 
максимальном жиме? Тут нужно сравнивать величины 
одного порядка. А вес 150 кг (1470 Н), уважаемый 
Составитель - это, между прочим, именно средняя сила, 
развиваемая при очень медленном неускоренном жиме 
150 кг. Убедиться в этом можно благодаря следующим 
нехитрым рассуждениям.

В самом начале движения скорость штанги равна нулю, в 
конце движения скорость штанги тоже равна нулю 
(медленный жим предполагает, что атлет не подбрасывает 
штангу после завершения движения, а завершает движение 
так, чтобы штанга сама остановилась в наивысшей точке 
своего подъёма). И это, между прочим, довольно близко 
к реальной картине предельного жима, где сил у атлетов 
зачастую хватает только на то, чтобы выпрямить руки - 
то есть штанга замирает в конце пути, не имея 
кинетической энергии (это при скоростном жиме штангу 
приходится останавливать в верхней точке и гасить 
набранную ею скорость), а иной раз штангу и вовсе не 
удаётся дожать. 

В таком случае можно воспользоваться законом 
сохранения энергии. 

В начале движения кинетическая энергия штанги равна 
нулю, в конце движения тоже - значит, работа, 
совершаемая мышцами (А), полностью пойдёт на изменение 
потенциальной энергии штанги в поле гравитационных 
сил. А изменение потенциальной энергии равно mgh, где 
m - масса штанги, g - ускорение свободного падения, 
h - высота, на которую поднялась штанга. Итак

A = mgh         (1)

С другой стороны, работа есть ничто иное, как 
произведение силы (F) на пройденное расстояние (h). В 
случае обычного жима сила на разных участках 
траектории различна, поэтому работа есть интеграл от 0 
до h по F(h) x dh, где  dh (дельта аш) - приращение 
пути, величина, по которой производится интегрирование,
а F(h) - функция зависимости силы от h (высоты 
подъёма).

Впрочем, тут можно обойтись и без интегрирования - 
если ввести понятие средней силы Fср (величину которой 
мы и ищем). В этом случае можно утверждать, что этот 
интеграл, а соответственно, и работа, равны 
произведению средней силы на расстояние. То есть 

А = Fср x h           (2)

Или из (1) и (2) получаем:

Fср x h = mgh

Сокращаем обе части упавнения на h и получаем:

Fср = mg

То есть получается именно то, о чём я и писал: при 
предельном переменнослабоускоренном жиме штанги 
средняя сила воздействия на штангу равна весу штанги.

Вот этих-то аргументов мне и показалось достаточно, 
чтобы сравнить среднюю силу при скоростном жиме со 
средней силой при предельном жиме 150 кг. Просто я 
иной раз публикую лишь результат моих рассуждений, а 
не сам их ход (когда это не имеет прямого отношения к 
рассматриваемой теме), да плюс к тому я забыл написать,
что сила 1470 Н при жиме 150 кг - это не просто 
абстрактная сила, а именно средняя сила, развиваемая 
при жиме 150 кг (честно сказать, я думал, что это и 
так понятно, я и до, и после того писал только о 
средних силах).

Я понял Вашу мысль про "потенциально возможную среднюю 
силу" (210-250 кг) - данная величина есть не что иное, 
как средняя сила статического напряжения мышц на всём 
протяжении траектории подъёма. И, конечно, с ней тоже 
можно сравнивать среднюю силу, развиваемую при 
скоростном жиме. Но реализовать такую силу, как Вы и 
пишете, по ряду причин невозможно. В итоге реальная, а 
не потенциальная средняя сила составляет всё те же 
1470 Н (вес 150 кг), о которых я и писал, и с которыми 
я сравнивал среднюю силу при скоростном жиме. 

Так что моё сравнение вполне правомерно и обдумано. 
Ваше же сравнение со средней статической силой лишь 
примиряет полученные мной результаты с соотношением 
Хилла и объясняет, почему разница в силе между 
медленным и скоростным жимом не столь существенна, как 
можно было бы ожидать из данного соотношения. 
Справедливости ради должен отметить: меня тоже 
несколько удивило, что средние силы отличались всего 
лишь в два раза - но я принял это как факт и не стал 
ломать над ним голову, поскольку прямого отношения к 
анализу толчка это уже не имело.
 
Ответить	Открыть подтему

Тема: Ещё немного критических замечаний
Автор: VadimPro
Дата: 30/03/2004 12:03
 	
Ответ на Ваши замечания, уважаемый Составитель, я
писал вчера вечером достаточно поздно, торопясь домой,
и потому успел лишь показать, что мои оценки силы были 
вполне правомерны. Но я не успел отметить, что в своих 
оценках Вы завысили среднюю силу при жиме 150 кг.

По Вашим расчётам, сила на всём пути, за исключением
"мёртвой" точки, должна была быть больше веса 150 кг и
опускаться до этой величины лишь в самой "мёртвой" 
точке. Так что мои 150 кг тут никак не 
"вырисовываются".

Однако по опыту я знаю, что нет ничего надёжней, чем 
расчёты на основе закона сохранения энергии, поэтому 
встаёт вопрос: в чем же Вы ошиблись?

Вы, похоже, завысили силы на этапах между стартом и
"мёртвой" точкой и после "мёртвой" точки. Вы даже 
назвали силу в "мёртвой" точке минимальной, то есть Вы 
полагали, что сила до достижения "мёртвой" точки была 
выше, чем сила в "мёртвой" точке - что, конечно же, 
совершенно не так.

Само название положения "мёртвая" точка означает, что 
сил атлета может не хватить для преодоления этого 
положения - однако сие вовсе не означает, что сила,
развиваемая в этом положении во время движения, 
минимальна. Нет, минимальной сила должна быть на каком-
то этапе приближении к "мёртвой" точке. Почему это 
так, объясню ниже.

При старте из состояния покоя развиваемая сила 
оказывается, конечно, выше, чем вес 150 кг (думаю, что 
Вы всё же слегка преувеличили данную силу, оценив её в 
220 кг; впрочем, истинная величина силы не столь уж и 
важна - то есть сила может быть и такой, как Вы 
предположили) именно благодаря тому, что сила рук 
превышает 150 кг. Вследствие этого штанга массой 150 кг
трогается с места и начинает  двигаться  даже с
некоторым ускорением, приобретая начальную скорость.

Однако стоит только штанге сдвинуться с места, как 
сила мышц сразу падает, и чем быстрее двигается 
штанга, тем значительнее снижается сила мышц. Сначала, 
понятно, сила снижается до веса штанги, и в этот 
момент штанга начиинает двигаться равномерно - с той 
самой скоростью, которую она приобрела на этапе 
разгона. Затем по каким-то причинам (усталость и/или
изменение соотношения рычагов) сила рук становится 
меньше веса штанги, и последняя, приближаясь к 
"мёртвой" точке, начинает уже терять свою скорость.

Само снижение скорости при приближении к "мёртвой" 
точке означает, что на штангу действует сила меньшая, 
чем вес штанги. В "мёртвой" точке штанга практически 
замирает и тут уже за счёт снижения скорости движения 
сила снова возрастает до величины веса штанги, и 
последняя очень медленно минует "мёртвую" точку. Тут
сила по причине изменения рычага различных мышц 
начинает возрастать и превышать вес штанги - за счёт 
чего штанга снова набирает небольшую скорость.

По мере набора скорости сила опять падает до веса
штанги, и если штанга завершает движение с этой же
скоростью, то сие значит, что сила была равна весу
штанги до самого конца движения. Если же штанга
останавливается в верхней точке, то это означает, что
на завершающем этапе сила снова опускается ниже веса
штанги. Таким образом, на всём протяжении относительно
равномерного жима штанги предельного веса сила,
развиваемая мышцами, колеблется в районе веса штанги -
потому-то среднее значение силы и равняется весу 
штанги.

В рассматриваемом мной и Вами случае жима штанги массой
150 кг вес штанги равнялся 1450 Ньютонам (вес - это 
сила, с которой тело действует на опору в 
гравитационном поле), и, соответственно, средняя сила, 
развиваемая мышцами, составляла 1470 Н.

Конечно, статическая сила рук в каждой точке траектории
штанги должна быть намного больше этой величины - тут 
Вы, уважаемый Составитель, правы. Но при жиме сила 
близка к статической только в самом начале и в 
"мёртвой" точке - на всём же остальном протяжении 
своего пути штанга движется с относительно высокой 
скоростью, и до реализации потенциала статической силы 
на этих участках уже далеко. Плюс к этому по мере 
медленного жима мышцы накапливают усталость и уже не 
могут реализовать ту силу, которую они могли бы 
развить в той или иной точке в своём свежем состоянии 
(как это происходит при оценке данной силы).

При скоростном же жиме 30 кг мышцы работают всего лишь 
в течение четверти секунды. Таким образом, в 
завершающей фазе скоростного жима штанга оказывается 
примерно через 0,25 сек после работы со средней силой, 
равной весу 76,5 кг. В случае жима предельного
веса в этой же точке штанга оказывается после работы 
мышц в течение несколько секунд со средней силой, 
равной весу 150 кг. Понятно, что в том состояние, в 
котором мышцы оказываются к моменту подъёма до 
указанной точки, они уже не могут развить свой 
теоретически возможный силовой потенциал.

Соотношение Хилла установлено для сокращения 
изолированных волокон, произведённого на разных 
скоростях, но при прочих равных условиях. В реальных 
же жимах с разной скоростью сильно различается общее 
время нахождения волокон под нагрузкой. Помимо 
простого соотношения Хилла в дело тут вмешивается
бОльшая усталость мышц при медленном и более 
длительном жиме штанги предельного веса - и в 
результате средние силы, развиваемые мышцами при 
движении с разной скоростью, отличаются уже не столь
существенно, как это можно было бы предположить из 
чистого соотношения Хилла.

Вы, уважаемый Составитель, были на правильном пути, но
немного ошиблись в своих оценках.
 
Ответить	Открыть подтему

Тема: Ещё меньше критических замечаний
Автор: Составитель
Дата: 02/04/2004 09:56
 	
Уважаемый Вадим, в своих последних ответах Вы 
отреагировали, увы, не совсем на то, на что я хотел 
обратить Ваше внимание в своём тексте "Эх, опубликую 
побыстрее..."

Вы уж извините меня за мой давешний ответ Вам по 
почте - я ещё поругаю себя за него в будущей рецензии. 
Но вот в оценке величины средней силы рук при 
медленном движении прав всё-таки я, а не Вы.

Про что Вы написали в своих ответах на "Эх, опубликую 
побыстрее..."? Про то, что при реальном жиме штанги в 
150 кг руки прикладывают к ней среднюю силу, близкую к 
той же самой величиине в 150 кг. Но я сие даже и не 
думал отрицать. Я писал совсем о другом: о жиме штанги
переменного веса (или жиме, выполняемом на некоем 
гипотетическом тренажёре, позволяющем предлагать рукам 
на каждум участке подъёма такую нагрузку, которую они 
на этом участке еле-еле могут преодолеть). Вот в этом 
случае и будет выявлена та сила, которая при чтении
Вашей статьи о силе рук при толчке от груди не вызовет 
недоуменных вопросов типа "А почему при столь 
значительном уменьшении скорости сокращения мышц их 
сила увеличивается столь незначительно?"

Что за силу при медленном жиме нужно выявить? Надо ли 
тут акцентировать внимание на явления усталости от 
долгого напряжения мышц или на относительно высокую 
скорость их сокращения в начале и конце подъёма
(используемую атлетом для его, подъёма, оптимизации)? 
Нет - ведь при быстром, мгновенном жиме явления 
усталости практически отсутствуют, не успевают 
возникнуть (следовательно, для корректного сравнения 
быстрого жима с м медленным нужно подбирать такие 
условия, в которых усталость почти отсутствует - 
например, по той причине, что в выраженном виде ещё 
просто не успевает наступить), а технически выгодные 
высокие скорости в начале и конце медленного подъёма 
постоянного веса в целом здорово уменьшают величину 
тей усилий, которую атлет реально может приложить на 
разных участках жима и, соответственно, здорово 
уменьшают среднюю величину его максимальной силы.

Может быть, для того, чтобы сделать сравнение 
медленных и быстрых жимовых усилий более корректным, 
не зависящим от смущающих Вас факторов, стоит 
рассматривать не полнообъёмные жимовые усилия, а
усилия именно на максимально коротких дистанциях? То 
есть тут нужно отвечать на такие вопросы: если при 
быстром (производимом за четверть секунды) жиме штанги 
стоя усилие рук в какой-то точке L общего подъёма
составляло N килограммов, то какую силу смогут 
приложить руки в этой же точке подъёма L при своём 
медленном распрямлении? Конечно, данный способ 
сравнения сил тоже не совсем корректен (для 
максимализации его корректности нужно замерить силы в 
возможно большем числе точек и всё равно найти некие 
средние значения усилий) - но, надеюсь, сама
нарисованная картина поможет Вам понять мою точку 
зрения на рассматриваемый вопрос.

Что же касается Вашего утверждения о том, что сила при 
жиме должна быть минимальной не в самой "мёртвой" 
точке, а именно на каком-то этапе приближения 
к "мёртвой" точке, то с ним можно было бы полностью 
согласиться - если бы не одно "но": упомянутый "этап 
приближения" настолько невелик, что для повышения 
ясности постановки вопроса, для увеличения простоты и 
т.д. величиной "этапа приближения" лучше пренебречь. 

Дело в том, что земная сила тяжести слишком уж здорово 
снижает высоту подлёта при низких скоростях подброса. 
Например, тело, имеющее вертикальную скорость порядка 
0,5 м/сек, может совершить свободный подлёт всего лишь 
на 1 см. То есть на протяжении этого одного сантиметра 
вроде бы вполне приличная для жима предельного веса
скорость в 0,5 м/сек (Ботев с такой скоростью почти 
полностью разогнёт свои руки всего лишь за секунду, в 
то время как неплохим тут будет и время жима порядка 
двух-трёх секунд) упадёт до нуля. Так что в реальности 
точка минимального усилия не совпадает с "мёртвой" 
точкой не более, чем на несколько миллиметров (а 
скорее всего, вообще даже на бесконечно малую 
величину).
 
Ответить	Открыть подтему

Тема: Re: Ещё меньше критических замечаний
Автор: VadimPro
Дата: 02/04/2004 19:28
 	
Уважаемый Составитель, я думал, что после моих 
последних разъяснений прения сторон по вопросу средних 
сил при медленном жиме штанги закончатся, однако же 
своим последним сообщением Вы ввергли меня в некоторое 
недоумение - в особенности, вот этими своими словами:

"Но вот в оценке величины средней силы рук при 
медленном движении прав всё-таки я, а не Вы."

Получается, что я, указав, точное значение средней 
силы при медленном жиме 150 кг (а средняя сила в этом 
случае равна весу 150 кг), а затем доказав это в 
расчётах, - не прав, а Вы, утверждая, что при жиме 
150 кг сила, равная весу 150 кг - это минимальная 
сила, а средняя сила значительно выше и равна 175 кг. 
правы? 

Вот только несколько цитат из  Вашего сообщения:

"То есть при вынесении оценки силе рук Ботева во время 
медленного жима Вадим исходил примерно из таких же 
неверных представлений, из каких когда-то исходил я 
сам, когда писал, что сила рук при толчке от груди в 
точности равна весу штанги, выжимаемой за четверть 
секунды. Мне в моём случае надо было к весу штанги 
прибавить ещё и ту силу, которая обеспечивает высокую 
скорость подъёма данной штанги, а Вадиму в его случае 
нужно было учесть величину не минимальной силы в 
150 кг, а все величины той силы (точнее, её среднюю 
величину), которую атлет прикладывает (или способен 
приложить) во время жима стоя 150 кг."

"В свою очередь, при медленном жиме стоя в начале 
движения руки прикладывают к грифу силу порядка 
двухсот килограммов, к середине движения эта сила 
уменьшается до ста пятидесяти пяти килограммов и затем 
опять нарастает до двухсот килограммов. Среднее усилие 
получается тут равным ста семидесяти пяти килограммам."

Как ещё можно воспринимать эти Ваши слова, как не 
утверждение того, что при жиме 150 кг развиваемая 
средняя сила больше веса 150 кг и равна весу в 175 кг? 

Эти процитированные мной утверждения, уважаемый 
Составитель, какой бы смысл Вы в них ни вкладывали - 
не верны. Тут уж одно из двух: либо у Вас имеют место 
быть неверные представления о том, что есть средняя 
сила, либо Вы завысили её значение.

Для того, чтобы понять это, давайте посмотрим, что 
будет со штангой весом в 150 кг, если приложить к ней 
среднюю силу, равную весу 175 кг.  

Закон сохранения энергии в общем случае будет 
выглядеть так.

W(o) + А = Mgh + W(k)   (1)

Где: W(o) - изначальная кинетическая энергия штанги, 
А - полезная работа, совершаемая атлетом при подъёме 
штанги, Mgh - изменение потенциальной энергии штанги в 
гравитационном поле (M - масса штанги, g - ускорение 
свободного падения, h - высота подъёма штанги), W(k) - 
кинетическая энергия штанги в конце пути.

Начальная кинетическая энергия штанги равна нулю, так 
как атлет начинает движение из состояния, в котором 
штанга покоится. Полезная работа А равна произведению 
средней силы, действующей на штангу F, и пройденному 
штангой пути h.

А = Fh      (2)

Тогда из (1) и (2) получается:

W(k) = Fh - Mgh = h(F-Mg)

Подставляем значения F = 1715 Н, M = 150 кг, 
g = 9,8 м/сек2 (ставлю без минуса, поскольку 
направление движения в формулах уже учтено), 
h = 0,55 м. 

Получается:

W(k) = 135 Дж.

Кинетическая энергия, как известно, равна 

W = (MV^2)/2

Отсюда конечная скорость штанги V = КОРЕНЬ(2W/M)

Подставив значение кинетической энергии и массу нашей 
штанги, получаю, что скорость штанги массой 150 кг 
после жима со средней силой, равной весу 175 кг, 
должна составлять 1,34 м/с, то есть быть выше, чем 
скорость штанги у Ботева после подрыва. После 
окончания жима штанга, движущаяся с такой скоростью, 
должна подлететь ещё на 9 см. Как видите, если 
прикладывать к штанге массой 150 кг среднюю силу в 
175 кг, то это уже будет мало похоже на предельный 
медленный жим - скорее, это будет скоростной посыл 
штанги. 

Так что при жиме штанги максимального веса средняя 
сила, развиваемая атлетом, равна весу штанги - это 
факт, который для меня был очевиден даже без 
каких-либо расчётов.

Что же касается "мёртвых" точек, то Ваши аргументы, не 
спорю, сильны. Признаюсь, о величине зон торможения я
не подумал. Однако же гравитация остановит штангу на 
пути в несколько миллиметров только в том случае, если 
сила воздействия атлета на штангу в "мёртвой" точке 
вдруг резко упадет до нуля - но тогда ведь сила, 
развиваемая атлетом в "мёртвой" точке будет равна 
нулю, а не весу в 150 кг, как Вы утверждали ранее. 

В реальности же картина совершенно иная: сила атлета, 
после того, как он двигал штангу равномерно на 
некотором участке (на участке равномерного движения 
сила в точности совпадает с весом штанги), не резко 
падает до нуля, а лишь чуть-чуть снижается ниже веса 
штанги, и штанга постепенно и на весьма длинном 
участке начинает терять скорость до самой "мёртвой" 
точки, в которой штанга практически останавливается - 
и только тогда сила атлета возрастает до веса штанги, 
и благодаря этому "мёртвая" точка успешно минуется. 

Уважаемый Составитель, я готов даже пойти на 
компромисс и употреблять вместо понятия ""мёртвая" 
точка" понятие "мёртвая зона", понимая под "мёртвой 
зоной" некий участок траектории (независимо от его 
длины), на котором происходит снижение скорости штанги 
от достигнутой при разгоне до, фактически, нуля. Но 
тогда нужно понимать, что чем меньше этот участок по 
длине, тем до меньшей величины на этом участке 
опускалась сила атлета.  

Да, действительно, при введения понятия "мёртвой зоны" 
можно утверждать, что в этой зоне сила, развиваемая 
мышцами, минимальна (при употреблении понятия 
""мёртвой" точки" этого уже утверждать нельзя), но 
ведь эта минимальная сила будет МЕНЬШЕ веса тела, а не 
равна ему, как изначально утверждали Вы. И чем короче 
будет эта "мёртвая зона", тем существеннее будет 
падение силы атлета ниже веса штанги в этой "мёртвой 
зоне". 

Именно из-за этого падения силы ниже веса штанги 
средняя сила и получается равной весу штанги (ведь на 
других участках сила была либо равна весу штанги, либо 
чуть его превышала).

Так что в чём была Ваша правота при оценке средних 
сил (и, понятно, моя неправота) я, увы, отказываюсь 
понимать.

А вот почему средние силы при предельном жиме и при 
очень скоростном жиме различаются всего в два раза, то 
я об этом уже написал в своём предыдущем сообщении, и 
на понимание этого меня сподвигли именно Ваши 
рассуждения о том, какие силы на участках движения 
потенциально возможны и почему они не достигаются. Но 
у меня сложилось такое впечатление, что Вы, уважаемый 
Составитель, и тут со мной в чём-то не согласны, вот 
только в чём - я до конца так и не разобрался. 

Вы ставите вопрос "Надо ли тут (при объяснении причин 
небольшого различия средних сил при медленных и 
скоростных жимах) акцентировать внимание на явления 
усталости от долгого напряжения мышц или на 
относительно высокую скорость их сокращения в начале и 
конце подъёма (используемую атлетом для его, подъёма, 
оптимизации)?" 

Вы отвечаете на него "нет", и тут же объясняете почему:

"ведь при быстром, мгновенном жиме явления усталости 
практически отсутствуют, не успевают возникнуть".

Чем ставите меня в тупик. Выходит, что хотя при 
скоростном жиме нет никаких явлений усталости, а при 
предельном жиме мышцы, наоборот, сокращаются на 
пределе слишком долго и из-за этого значительно теряют 
силу на большей части амплитуды, в результате чего 
средняя сила при медленном жиме становится существенно 
меньше, чем она могла бы быть при отсутствии явления 
усталости - но учитывать эти различия при объяснении, 
почему средняя сила при медленном жиме лишь вдвое 
превысила среднюю силу при скоростном жиме тем не 
менее не нужно?

Как это понимать? Впервые за всё время общения с Вами 
у меня складывается впечатление, что либо я совершенно 
не понимаю того, что Вы хотите мне сообщить, либо же 
Вы совершенно не поняли то, о чём писал я. Так что 
давайте устранять это недопонимание.
 
Ответить	Открыть подтему

Тема: Совсем мало критических замечаний
Автор: Составитель
Дата: 02/04/2004 20:58
 	
Уважаемый Вадим, Вы, возможно, опять куда-то 
торопились (поэтому, видимо, в процессе своих 
вычислениях параметров движения штанги в 150 кг почему-
то решили узнать её финальную скорость при длине 
разгона в полные 55 см, а не в начальные 10-15 см) и 
снова начали писать о разных второстепенных вещах (всё 
о той же неувядающей штанге в 150 кг), а не о том, на 
что я пока безуспешно пытаюсь обратить Ваше внимание 
(например, на тренажёр с переменным сопротивлением на 
разных высотах подъёма). 

Пожалуйста, забудьте о реальном жиме штанги в 150 кг - 
он даёт не истинное, а уменьшенное представление о 
силовом потенциале рук атлета, способного в течение 
четверти секунды прикладывать к штанге среднюю силу в 
76,5 кг. (Вполне допускаю, что с оценкой явлений, 
происходящих при реальном жиме, я кругом неправ - 
хотя, скорее всего, это не так).

Для выявления истинного силового потенциала рук атлета 
(напоминаю: выявление этого потенциала нужно для того, 
что Ваша статья про силу рук при подъёме от груди не 
имела слабых, вызывающих недоумение мест) надо 
отбросить всё, ставящее силы рук при быстром и при 
медленном движениях в неравные условия. 

Если более-менее сведущий в вопросах приложения силы 
человек прочитает в Вашей статье, что при разгибании 
чуть ли не на пределе скорости руки прикладывают 
среднюю силу, всего лишь вдвое меньшую, чем при жиме с 
остановкой в "мёртвой" точке - то он точно заподозрит 
здесь какой-то просчёт. Этот человек чисто 
автоматически подумает, что при медленном жиме руки 
полностью реализуют свой "медленно-двигательный" 
силовой потенциал. И этому человеку надо объяснить, 
что на самом деле при жиме с остановкой в "мёртвой" 
точке "медленно-двигательный" силовой потенциал 
реализуется далеко не полностью. В то время как при 
толчке от груди, позволяющем рукам приложить за 
четверть секунды среднюю силу в 76,5 кг, происходит 
почти полная реализация "быстро-двигательного" силового
потенциала (в удобных точках здесь прикладывается 
значительная сила, а неудобные точки пролетаются на 
самой высокой скорости).

Вы задали мне следующий вопрос:

"Выходит, что хотя при скоростном жиме нет никаких 
явлений усталости, а при предельном жиме мышцы, 
наоборот, сокращаются на пределе слишком долго и из-за 
этого значительно теряют силу на большей части 
амплитуды, в результате чего средняя сила при 
медленном жиме становится существенно меньше, чем она 
могла бы быть при отсутствии явления усталости - но 
учитывать эти различия при объяснении, почему средняя 
сила при медленном жиме лишь вдвое превысила среднюю 
силу при скоростном жиме тем не менее не нужно?"

Ну конечно, не нужно. При организации честных 
соревнований противников надо ставить в максимально 
равные условия. Поэтому если поставлена задача выявить 
истинные силы противников, но один из них устал, то 
ему следует дать отдохнуть. Если же условия 
соревнований такие, что в них будет выявляться 
максимальная сила соперников, проявляемая в 
максимально удобных для них условиях, и при этом 
первый участник соревнований привык поднимать штангу 
на бегу, а второй - в полной неподвижности, то второго 
участника не надо заставлять поднимать штангу на ходу: 
ведь требуется выявить именно максимальные силы 
противников.

В конце концов, уважаемый Вадим, попробуйте всё-таки 
принять предлагавшуюся мной ранее точку зрения с 
измерением силы рук, разгибаемых с разными скоростями 
на очень коротких дистанциях. Уж здесь-то превышение 
медленной силы над быстрой будет выявлено почти в 
чистом виде и наверняка окажется всё тем же 
трёхкратным (как минимум).

Что же касается месторасположения "мёртвых" точек, то 
в этом вопросе всё зависит от величины, от уровня 
падения силы при приближении к "мёртвой" точке. Если 
величина падения силы большая (крайний вариант - сила 
резко падает до нуля), то предварительный разгон 
поднимет снаряд заметно выше точки максимального 
падения силы. Если же величина падения силы маленькая 
(крайний вариант - до "мёртвой" точки Т сила вообще не 
падает), иными словами, подъём на достаточно длинном 
(относительно величины подлёта за счёт имеющейся 
скорости) участке становится близким к 
равнозамедленному, то в конце концов штанга может 
окончательно замедлиться и остановиться как раз в 
"мёртвой" точке Т. 

Поскольку реальность ближе к первому варианту, то в 
нашем споре о месторасположении "мёртвых" точек более 
правы Вы.
 
Ответить	Открыть подтему

Тема: Точки над "ё" (надеюсь)
Автор: VadimPro
Дата: 06/04/2004 13:36
 	
Увы, уважаемый Составитель, похоже, поторопились в 
данном случае именно Вы - поскольку Вы так и не 
заметили, что же именно я написал Вам двумя первыми 
ответами на Ваши критические замечания. Если бы Вы 
поняли мои слова, то продолжения дискуссии по этому 
вопросу уже не потребовалось бы. 

Вы написали о том, что я не на то обращаю внимание в 
Ваших сообщениях, и призывали меня задуматься над тем 
тренажёром, о котором упомянули в первом своём 
послании ("Эх, опубликую побыстрее, пока не 
опередили..."). Но я ведь ещё прямо в первом же своей 
реакции на данное сообщение написал, что прекрасно 
понял эту Вашу мысль и обратил на неё самое серьёзное 
внимание.

Да, если измерить статическую силу в каждой точке 
траектории, а затем вычислить среднее значение такой 
силы, то для атлета, чей максимальный жим 150 кг, 
такая сила будет выше, чем вес 150 кг. Но дело в том, 
что данная величина не есть средняя сила при жиме 
150 кг - лишь некая величина, рассчитанная путём 
усреднения максимальных статических сил на всей 
амплитуде движения. Прямого отношения к реальному жиму 
150 кг она не имеет и уж тем более не является средней 
силой, развиваемой при жиме 150 кг. Вы же, повторяю, 
сами в своем сообщении призвали сравнивать среднюю 
силу при скоростном жиме именно со средней силой при 
максимальном жиме. Но ведь именно так я и поступил, 
сравнив силу при скоростном жиме с жимом 150 кг: я 
сравнил среднюю силу при скоростном жиме со средней 
силой при предельном жиме (то, что это так, я, надеюсь,
уже доказал). Предложенная же Вами для сравнения 
величина не есть средняя сила. Это не значит, конечно, 
что с Вашей величиной среднюю силу при быстром жиме 
нельзя сравнивать (нет, это вполне возможно, и даже 
любопытно), но только это не средняя сила, понимаете? 

Как это ни удивительно, но средняя сила при скоростном 
жиме отличается от средней силы при предельном жиме 
всего лишь в два раза. Как всё это соотносится с 
формулой Хилла, позволяют понять Ваши рассуждения о 
потенциально возможной статической силе на разных 
участках амплитуды. А почему они не реализуются, я 
написал ещё в первом ответе на Ваши замечания. Почему 
Вы после этого опять заявили, что я неправ в расчёте 
средних сил - я, увы, никак не могу понять. 

Похоже, что Вы всё-таки изначально действительно 
полагали, что рассчитанная Вами сила и есть средняя 
сила, развиваемая при жиме 150 кг, а сила, равная весу 
150 кг - это минимальная сила при жиме 150 кг: Вы об 
этом написали прямым текстом. 

Я бы ещё понял Вашу позицию, если бы Вы утверждали не 
то, что я неверно посчитал среднюю силу, а то, что я 
проводил сравнения вообще не с тем параметром, что 
нужно сравнивать не со средней силой при предельном 
жиме, а как раз с усреднённым по всей амплитуде 
максимальным значениям статической силы. Но тогда это 
же совсем другой вопрос, и я ещё в первом своём ответе 
написал, что такое сравнение возможно, и что оно лучше 
позволяет понять, как полученные мной данные 
увязываются с соотношением Хилла. Но это вовсе не 
отменяет правомерность моего сравнения со средней 
силой (тем более, что формально Вы сами именно к этому 
и призывали). А почему я сравнивал именно со средней 
силой? Дело в том, что именно средняя сила нам более-
менее известна - в отличие от рассматриваемой Вами 
средней величины максимально возможного усилия. Массы 
штанги, поднимаемые атлетами в жиме, хорошо известны, 
а вот прибор предложенный Вами, пока никто не 
сконструировал, и замеры статических сил на всём 
участке траектории никто не проводил. Как можно 
сравнивать с такой весьма неопределённой величиной? 

Вот Вы написали, что в начале движения сила равна 
220 кг. А почему не 190 кг или не 250 кг? Кто это 
проверял? Зато то, что атлеты такого уровня, как 
Ботев, могут выжать что-то около 150 кг - это уже 
более-менее надёжная опора для сравнения, потому-то я 
ею и воспользовался. 

Но удивило, меня, повторяю, не то, что Вы предложили 
другое сравнение, а то, что после всех моих 
разъяснений Вы опять стали утверждать, что я не прав в 
вопросе средних сил.

Более того, и в своём последнем сообщении Вы опять 
продолжаете упорствовать, я опять не понимаю что 
значат вот эти Ваши слова:

"Уважаемый Вадим, Вы, возможно, опять куда-то 
торопились (поэтому, видимо, в процессе своих 
вычислениях параметров движения штанги в 150 кг почему-
то решили узнать её финальную скорость при длине 
разгона в полные 55 см, а не в начальные 10-15 см)"

Сии слова свидетельствуют, что Вы всё-таки 
недостаточно чётко осознаёте, что такое средняя сила. 

Представьте себе некую функцию f по некоему аргументу 
x, то есть у = f(x) - пусть данная функция 
представляет собой некую волнистую линию лежащую над 
осью х. Отметьте на оси х некий отрезок от x1 до x2. 
Теперь заштрихуйте площадь, лежащую между графиком 
функции f(x) и осью x на участке от x1 до x2. Так вот, 
среднее значение функции f(x) на участке от x1 до x2 
есть такая прямая, при которой площадь, лежащая между 
осью x и данной прямой на участке от x1 до x2, 
численно равняется площади под волнистой функцией 
f(x). Это, по сути, и является определением такой 
величины, как среднее значения функции.

Если по оси x откладывать высоту подъёма штанги h, а 
f(h) понимать как зависимость силы от высоты подъёма, 
то заштрихованная площадь будет не чем иным, как общей 
работой силы, воздействующей на штангу, совершённой 
при подъёме снаряда. Сложную фигуру под волнистой 
линией можно разбить на маленькие прямоугольнички, и 
поскольку площадь прямоугольника равняется произведению
сторон, то есть dh умножить на df - сила, умноженная 
на расстояние, то это и есть работа силы на этом 
маленьком участке перемещения. А сумма всех 
прямоугольников, участвующих в разбиении, - это, 
соответственно, общая работа силы на участке от x1 до 
x2.

Таким образом, средняя сила - это такая сила, при 
действие которой общая работа, совершенная на всём 
участке перемещения, будет равна работе переменной 
силы. 

Всё вышеизложенное я привёл для того, чтобы показать 
физический смысл средней силы и то, что при 
использовании средних сил правомерно применение закона 
сохранения энергии (так как в нём мы оперируем как раз 
совершённой работой, вычисление которой возможны как 
по переменной, так и по средней силе)- но только на 
том участке траектории, на котором и вычислялась 
средняя сила. Таким образом, зная среднюю силу, 
действующую на определённом участке, можно рассчитать 
только энергии штанги и, соответственно, скорости к 
концу этого участка. Расчёт же скоростей в 
промежуточных точках траектории на основе средних сил 
лишён какого-либо физического смысла - ведь работа на 
произвольно взятом участке может быть выше или ниже 
чем работа, рассчитанная на основе средней силы и 
перемещения. 

Работа, рассчитанная на основе средних сил, совпадает 
с реальной только при расчёте работы на том же 
участке, на котором и вычислялась средняя сила. В 
случае жима 150кг и известной средней силы (я 
утверждаю, что она равна весу 150 кг, Вы - что она 
равна весу 175 кг), мы можем посчитать только скорость 
штанги в конце движения, и никаких расчётов скоростей 
на высоте 10-15 см сделать по этой характеристике 
(средней силе) невозможно - это будет бессмыслица. 
Чтобы посчитать скорость в предложенных Вами точках 
траектории, нужно знать либо точные значения силы на 
этих участках траектории, либо среднюю силу, но уже 
именно на этом участке, которая, повторюсь, нам 
априори не известна, поскольку никто её не измерял. Мы 
знаем только массу штанги, которую может поднять 
атлет, а следовательно, и среднюю силу на всей 
амплитуде подъёма, равную весу штанги. Да и вообще к 
чему эти расчёты на 10-15 см, что они должны дать? 

Я-то проводил свои расчёты для того, чтобы показать, 
что если атлет выжимает 150 кг на пределе своих 
возможностей, то развиваемая им средняя сила никак не 
может быть равна весу 175 кг. Если же эти данные нужны 
для того, чтобы сравнить значение силы на этом участке 
со скоростным жимом, тогда и для скоростного жима 
нужно брать не среднее значение силы, а силу на этом 
же участке - но она, как Вы могли убедиться по расчёту 
толчка Ботева, на этом участке тоже значительно выше, 
чем в среднем при толчке.

В своем последнем сообщении на мой вопрос, нужно ли 
при сравнении скоростного жима и предельного жима 
учитывать различия в усталости мышц, Вы ответили 
следующее:

"Ну конечно, не нужно. При организации честных 
соревнований противников надо ставить в максимально 
равные условия. Поэтому если поставлена задача выявить 
истинные силы противников, но один из них устал, то 
ему следует дать отдохнуть. Если же условия 
соревнований такие, что в них будет выявляться 
максимальная сила соперников, проявляемая в 
максимально удобных для них условиях, и при этом 
первый участник соревнований привык поднимать штангу 
на бегу, а второй - в полной неподвижности, то второго 
участника не надо заставлять поднимать штангу на ходу: 
ведь требуется выявить именно максимальные силы 
противников."

Данные Ваши слова опять демонстрируют непонимание 
моего текста и обращённого к Вам вопроса. Я не 
размышлял о том, как и с чем лучше проводить сравнение 
сил при быстром и при медленном жимах. Я не задал 
вопрос, нужно ли проводить сравнение так, чтобы в нём 
пришлось учитывать усталость мышц - вопрос, на который 
бы Вы могли ответить: "Ну конечно, не нужно". 

Нет, я уже провёл сравнение средней силы при 
скоростном жиме и средней силы при предельном жиме 
(почему я это сделал, я объяснял выше - нам хорошо 
известны только средние силы при предельном жиме, но 
никак не точное распределение сил по всей амплитуде). 
В итоге я получил, что средние силы при скоростном и 
предельном жимах отличаются всего в два раза. Так вот, 
при объяснении того, почему данный факт имеет место 
быть (или Вы по-прежнему не согласны, что средние силы 
отличаются всего в два раза - но никак не в три или 
больше?), я среди прочих объяснений привёл в качестве 
аргумента ещё и гораздо большую усталость мышц (а 
следовательно, и большее снижение развиваемой силы) 
при предельном жиме. В связи с этим Ваш ответ "Нет" 
выглядит как нежелание признать тот факт, что 
усталость приводит к снижению силы.

Далее уважаемый Составитель, Вы написали:

"В конце концов, уважаемый Вадим, попробуйте всё-таки 
принять предлагавшуюся мной ранее точку зрения с 
измерением силы рук, разгибаемых с разными скоростями 
на очень коротких дистанциях. Уж здесь-то превышение 
медленной силы над быстрой будет выявлено почти в 
чистом виде и наверняка окажется всё тем же 
трёхкратным (как минимум)."

Я на всякий случай ещё раз повторяю, что Вашу точку 
зрения именно в этом вопросе я принял, понял и одобрил 
давно и сразу, и даже написал об этом в первом же 
ответе. Именно потому мне столь удивителен и был Ваш 
последующий ответ.
 
Ответить	Открыть подтему

Тема: Re: Точки над "ё" (надеюсь)
Автор: Чумбурмучка
Дата: 07/04/2004 22:43
 	
Многоуважаемые участники дискусси, позвольте и мне 
внести свои три копейки в разговор. Собственно, теперь 
уже я кое-что не понимаю.

Итак, реальная сила при жиме штанги весом 150 кг есть 
некоторая функция высоты подъёма. Пусть на графике на 
оси абсцисс мы откладываем высоту подъёма штанги, при 
этом x1 - начальное положение штанги, x2 - конечное 
положение. По оси ординат мы откладываем силу, 
действующую на штангу при перемещении из x1 в x2. 
Допустим, что общий вид графика силы нам неизвестен, 
но известно, что минимум графика расположен в точке 
1470 Н (150 кг), где x равно "мёртвой" точке. Таким 
образом, все остальные точки графика лежат не ниже 
данной. Площадь между графиком реальной силы и осью 
абсцисс есть общая работа силы.

При предполагаемой Вадимом средней силе, равной 1470 Н,
её график представляет собой прямую, идущую из точки 
x1 в x2 и параллельную оси абсцисс. Эта прямая средней 
силы касается линии реальной силы в минимальной точке, 
которую я упомянул выше. То есть при этом все точки 
прямой средней силы лежат не выше линии реальной силы.

Теперь посмотрим на площади. Площадь под прямой средней
силы должна быть равна площади под линией реальной 
силы. Но если все точки прямой средней силы лежат не 
выше линии реальной силы, то это возможно только в том 
случае, если линии реальной силы и средней силы 
совпадают.

Либо что-то здесь не так, либо я чего-то не учёл.
 
Ответить	Открыть подтему

Тема: Уважаемый Вадим, Ваши надежды оправдались
Автор: Составитель
Дата: 07/04/2004 23:10
 	
Уважаемый Вадим, в нашем с Вами споре о средней силе 
при жиме 150 кг правы всё-таки Вы, а не я. И в своём 
первом сообщении ("Эх... и т.д.") я совершенно 
напрасно написал, что Вы неверно оценили среднюю силу 
при жиме ста пятидесяти килограммов - она 
действительно составляет ровно 150 кг. При оценке этой 
средней силы передо мной снова стоял какой-то 
психологический барьер, какое-то антифизическое, 
мистическое представление о ситуациях с подъёмом 
штанги. 

Но и Вы тоже не могли сразу объяснить реальное 
положение дел попроще - всё писали какие формулы с 
какими-то ужасными дэ икс по дэ тэ и чуть было даже не 
начали что-то там интегрировать. А нужно было сходу 
написать: сила есть работа, делённая на расстояние. 
Работа равна потенциальной энергии поднятой штанги, 
расстояние равно длине выпрямленной кверху руки, 
следовательно, как ни хитри со скоростями подъёма, как 
ни меняй прилагаемую к штанге силу, она в итоге всегда 
окажется в среднем равной делённой на расстояние 
потенциальной энергии.

А вот с моей мыслью про тренажёр, позволяющий извлечь 
из рук почти весь их силовой потенциал и тем самым 
серьёзно увеличить величину средней силы при подъёмном 
движении, Вы в самом первом ответе мне отнюдь не были 
согласны.

Вы написали следующее:

"Я понял Вашу мысль про "потенциально возможную 
среднюю силу" (210-250 кг) - данная величина есть не 
что иное, как средняя сила статического напряжения 
мышц на всём протяжении траектории подъёма. И, 
конечно, с ней тоже можно сравнивать среднюю силу, 
развиваемую при скоростном жиме. Но реализовать такую 
силу, как Вы и пишете, по ряду причин невозможно. В 
итоге реальная, а не потенциальная средняя сила 
составляет всё те же 1470 Н (вес 150 кг), о которых я 
и писал, и с которыми я сравнивал среднюю силу при 
скоростном жиме."

Реализовать такую силу (хотя бы в описанном тренажёре) 
при большом желании всё-таки можно. Если преодолеть 
трудности конструирования и изготовления описанного 
тренажёра - то можно. Человек будет, медленно двигая 
рычаг тренажёра, прикладывать сначала силу 220 кг, 
потом 200 кг, потом 180 кг и т.д. до величины, 
допустим, 130-140 кг, а затем сила воздействия его рук 
начнёт постепенно нарастать до величины в 250 кг и 
более (возможно, на каких-то участках человек будет 
получать специальные небольшие послабления - поскольку 
тренажёр, допустим, "умный"). Так что средняя сила при 
таком "разносильном" жиме составит величину, 
существенно большую, чем 150 кг. Однако вместо 
реального согласия с этой картиной Вы опять завели в 
том ответе старую песню про жим ста пятидесяти 
килограммов.

Кстати, вроде бы в одном из сообщений в этой ветке Вы 
написали про такую интересную особенность жима лёжа, 
что из-за страшного уставания рук в процессе жима 
некоторые атлеты не находят сил дожать штангу, 
окончательно выпрямить уже почти совсем прямые руки. 
То есть вот ведь какая страшная усталость имеет место 
в процессе подъёма и вот ведь куда от этой страшной 
усталости перемещается "мёртвая" точка.

Уважаемый Вадим, у описанных Вами некоторых 
атлетов "мёртвая" точка смещается высоко вверх от 
положенного ей места вовсе не по причине страшной 
усталости, а по причине использования домкратоподобной 
экипировки. Жимовая майка сама поднимает штангу 
(Скотту Мендельсону его майка, как утверждает Юрий 
Устинов, "не дала положить на грудь" уже целых 420 кг) 
до района (или даже выше) традиционного положения 
"мёртвой" точки, и свою реальную силу атлету 
приходится прикладывать уже лишь в самом конце дожима. 
Если этой силы на подъём штанги не хватает, а майка 
уже перестаёт помогать, то тут-то и возникает новая и 
удивительная "мёртвая" точка.

А в обычном, "недомкратизированном" жиме "мёртвые" 
точки обитают по-прежнему по своим старым адресам.
 
Ответить	Открыть подтему

Тема: У нашей "ё" точек будет больше, чем обычно
Автор: VadimPro
Дата: 08/04/2004 11:57
 	
Уважаемый Составитель, я рад, что ситуация наконец-то 
проясняется, и в принципе эту часть дискуссии можно 
было бы закончить. Но я решил, что для того, чтобы всё 
окончательно прояснилось, нам нужно ещё немного 
"повыяснять отношения".

Вы написали:

"Но и Вы тоже не могли сразу объяснить реальное 
положение дел попроще - всё писали какие формулы с 
какими-то ужасными дэ икс по дэ тэ и чуть было даже не 
начали что-то там интегрировать. А нужно было сходу 
написать: сила есть работа, делённая на расстояние. 
Работа равна потенциальной энергии поднятой штанги, 
расстояние равно длине выпрямленной кверху руки, 
следовательно, как ни хитри со скоростями подъёма, как 
ни меняй прилагаемую к штанге силу, она в итоге всегда 
окажется в среднем равной делённой на расстояние 
потенциальной энергии."

Все верно, именно это я Вам и объяснял, только своим 
языком (он для меня столь же понятен, как и эти Ваши 
слова). Единственное, что к Вашим словам нужно 
добавить - это будет иметь место только в том случае, 
если в начале движения и в конце движения штанга не 
имеет скорости (нет начальной и конечной кинетической 
энергии). Но мои объяснения с приращениями аргумента и 
функции (в данном случае высоты подъёма и силы рук) 
позволяют ещё дополнительно понять, что определение 
таким образом средней силы есть не просто формальное 
определение. Нет, полученная таким образом средняя 
сила есть действительно та средняя сила, которую мы 
себе и представляем, говоря о средней силе, 
действовавшей при подъёме штанги. Она представляет 
собой, по сути, сумму (интегрирование) произведений 
реальной, действовавшей силы на том маленьком кусочке 
пути, на котором данная сила действовала (де аш), 
делённую на общий пройденный путь.

И ещё. Я приводил свои вычисления, чтобы максимально 
точно объяснить ситуацию - ведь средняя сила строго 
равна весу штанги только в том случае, если конечная 
скорость штанги равна нулю. Но тогда ведь можно 
утверждать, что в конечной точке штанга имеет 
некоторую скорость, а следовательно, и энергию, а 
значит, средняя сила была выше веса штанги на 
величину, необходимую для сообщения штанге этой 
кинетической энергии. Вот я и показал, что превышение 
средней силы над весом штанги пропорционально квадрату 
конечной скорости, а именно - (MV^2)/2h. То бишь если 
V стремится к нулю, то величина, пропорциональная 
квадрату скорости, тем более стремится к нулю. Или, 
если предположить, что средняя сила превышала вес 
штанги всего лишь на вес 25 кг, то есть была равна 
175 кг, а не 150 кг, то штанга в конце движения 
вылетала бы со скоростью 1,3 м/с. Именно эти аргументы 
должны объяснить, почему можно утверждать, что при 
предельном жиме средняя сила очень близка к весу 
штанги. 

Теперь о Вашем гипотетическом тренажёре. Вы написали:

"Реализовать такую силу (хотя бы в описанном 
тренажёре) при большом желании всё-таки можно. Если 
преодолеть трудности конструирования и изготовления 
описанного тренажёра - то можно. Человек будет, 
медленно двигая рычаг тренажёра, прикладывать сначала 
силу 220 кг, потом 200 кг, потом 180 кг и т.д. до 
величины, допустим, 130-140 кг, а затем сила 
воздействия его рук начнёт постепенно нарастать до 
величины в 250 кг и более (возможно, на каких-то 
участках человек будет получать специальные небольшие 
послабления - поскольку тренажёр, допустим, "умный"). 
Так что средняя сила при таком "разносильном" жиме 
составит величину, существенно большую, чем 150 кг. 
Однако вместо реального согласия с этой картиной Вы 
опять завели в том ответе старую песню про жим ста 
пятидесяти килограммов."

Естественно, я завёл песню о том, почему при жиме 
реальной штанги весом 150 кг те силы, о которых Вы 
упомянули, не реализуются. Понимаете? Тут и спорить-то 
было не о чем. У нас с Вами есть предельно возможный 
жим штанги весом 150 кг, в ходе которого, как мы 
установили, развивается средняя сила всего в два раза 
большая, чем при скоростном жиме, что вроде бы не 
очень согласуется с соотношением Хилла. Вопрос: почему?
Ответ - да потому, что в соотношении Хилла верхним 
пределом, с которым всё сравнивается, является 
статическая сила. А статическая сила на всей 
траектории движения штанги значительно выше, чем вес 
150 кг, но при реальном предельном жиме штанги такая 
статическая сила не реализуется, поскольку штанга 
всё-таки движется с некоторой отличной от нуля 
скоростью. 

И даже очень отличной: допустим, предельный жим длится 
4 сек, а штанга поднимается на 0,6 м - тогда средняя 
скорость движения штанги равна 0,15 м/сек, а это уже 
далеко не статическое напряжение мышц, и подъёмная 
сила в этом случае начинает отличаться от статической 
уже довольно существенно. Да, к тому же, за эти 4 сек 
сила мышц существенно снизится из-за усталости, и к 
концу движения такие мышцы даже при статическом 
напряжении будут неспособны реализовать ту статическую 
силу, которая измерялась в данной точке траектории при 
неуставших мышцах. 

Более того - поставь мы на штангу больший вес, мышцы 
смогли бы выжать его благодаря более медленному 
движению только на начальной части амплитуды - но 
тогда не смогли бы преодолеть "мёртвую" точку. То есть 
я изначально был согласен с этой частью Ваших замечаний
- ведь, как мне показалось, Вы сами обо всём об этом 
написали. Я просто отметил, что эти Ваши аргументы о 
статических силах могут объяснить, почему то, что 
средняя сила, при предельном жиме равная всего лишь 
150 кг, не противоречит соотношению Хилла.

Но мои последние слова вовсе не означают, что я 
согласен с тем, что ту среднюю статическую силу, о 
которой Вы написали, можно реализовать и в придуманном 
Вами тренажёре. Дело в том, что очень близко 
реализовать статическую силу можно только в том 
случае, когда скорость штанги реально стремится к 
нулю. Допустим, при жиме со скоростью 0,001 м/с 
скорость можно считать близкой к нулевой и силу, 
реализуемую при такой скорости, близкой к статической 
(той, которую мы измеряли на всех точках подъёма). 
Если появляется тренажёр, не дающий двигаться быстрее 
или медленнее, чем с такой скоростью, то есть 
добавляющий или снижающий нагрузку при изменении 
скорости, то в этом случае жим штанги продлится 10 
минут. Как Вы думаете, что будет с силой мышц атлета 
при таком жиме к его концу? Реализует ли он свою 
среднюю статическую силу? 

Даже если предположить, что на скорости 0,01 м/сек 
реализуется сила, достаточно близкая к статической 
(хотя это уже далеко не так), то и в этом случае жим 
будет длиться 1 минуту. При непрерывном максимально 
возможном напряжении мышц в течение минуты сила мышц 
может упасть более чем вполовину, так что до 
реализации той силы, о которой Вы писали, тут тоже 
далеко. Если же Вы собираетесь жать в данном тренажёре 
не долее 5-6 секунд (пока существенно не истощились 
запасы креатинфосфата в мышцах), то с такой скоростью 
жима не о каком приближении к статической силе даже и 
говорить не приходится. Так что описанная Вами 
предельно возможная сила не реализуема принципиально 
ни в каком тренажёре. 

Хотя, безусловно, средняя сила, развиваемая мышцами в 
предложенном Вами устройстве, будет значительно выше, 
чем вес, поддающейся подъёму штанги (но это всё равно 
будет ещё не усреднённая статическая сила). Так что 
усреднённая статическая сила - это нереализуемый 
ориентир, на который правильно будет ориентироваться 
только в соотношении Хилла.

Что же касается того, что дожать штангу не получается 
не по вине усталости мышц, а из-за использования 
экипировки, то тут я с Вами тоже не согласен. 

Нет, конечно, при использовании экипировки возникает 
именно такой эффект, о котором Вы написали, но это не 
значит, что при действительно предельном жиме в конце 
движения штанга вылетает со скоростью, заметно 
отличной от нуля. 

Я никогда не жал в экипировке и знаю, что иной раз 
после того, как поборешься со штангой в "мёртвой" 
точке, сил действительно хватает только на то, чтобы 
выпрямить руки и зафиксировать штангу. Так что падение 
силы из-за усталости при предельных жимах - это 
реальность, а не мой вымысел.
 
Ответить	Открыть подтему

Тема: Ещё раз о "мёртвых" точках
Автор: VadimPro
Дата: 08/04/2004 13:09
 	
Уважаемый Чумбурмучка, Ваши рассуждения совершенно 
логичны, но Вы, к сожалению, допустили ошибку ещё на 
этапе постановки задачи. 

Приняв за данность, что при жиме минимальная сила 
достигается в "мёртвой" точке и что равна она якобы 
весу 150 кг, Вы тем самым заложили в условие задачи 
противоречие, с которым в конце концов и столкнулись. 
Я уже писал об этом Составителю, и повторяю Вам: 
минимальная сила при жиме развивается не в "мёртвой" 
точке, а до неё - в так называемой "мёртвой зоне". И 
уже само снижение скорости при подходе к "мёртвой" 
точке свидетельствует о том, что сила, действующая на 
штангу на этом этапе, меньше её веса - то есть меньше 
150 кг. Реальный график сил выглядит несколько иначе, 
чем Вы его себе представили.

При старте сила лежит выше прямой линии средней силы и 
постепенно по мере набора штангой скорости к ней 
приближается (снижение силы происходит тут в 
результате увеличении скорости движения). В точке, где 
набор штангой скорости заканчивается (а это значит, 
что сила сравнялась с весом штанги), линия силы 
касается прямой средней силы (являющейся одновременно 
ещё и весом штанги), затем, если некоторое время 
скорость жима не меняется, реальная сила совпадает со 
средней. Далее из-за изменения плеч рычагов сила 
падает ниже веса штанги и штанга начинает терять 
скорость - при этом график силы, понятно, идёт уже под 
прямой средней силы и где-то на этом участке достигает 
точки минимума. При этом из-за снижения скорости 
движения сила начинает расти (несмотря даже на 
"невыгодность" соотношения плеч рычагов), а значит, 
график силы приближается снизу к прямой линии средней 
силы (веса штанги). При этом сама штанга всё равно ещё 
теряет скорость, поскольку сила рук меньше веса 
штанги. И в момент максимального приближения скорости 
штанги к нулевой отметке (реально это всё равно некая 
очень маленькая скорость) сила, развиваемая атлетом, 
поднимается до величины в 150 кг и, следовательно, 
линия силы касается прямой средней силы (веса штанги), 
но уже снизу. Далее, пока не минуется "мёртвая" точка, 
линии силы и средней силы идут некоторое время вместе 
и после прохода  "мёртвой зоны" сила мышц опять 
начинает расти, штанга набирает скорость, а линия силы 
поднимается над прямой средней силы (веса штанги). 
Затем из-за набора скорости сила сокращения мышц опять 
снижается и набор скорости приостанавливается - в этот 
момент сила опять опускается до веса 150 кг и 
сливается с прямой линией. 

Пока скорость не меняется, сила равна весу штанги, то 
есть линия силы и средняя линия идут вместе, затем, 
если в конечной точке штанга останавливается, то это 
означает, что где-то на финальном участке штанга 
теряла скорость, а следовательно, сила воздействия 
была ниже веса штанги. То есть в финальной фазе 
движения линия силы опять опустилась ниже средней силы 
(веса штанги). 

Таким образом, если посчитать площади под кривой 
линией реальной силы и прямой линией средней силы, то 
эти площади совпадут (сумма площадей под кривой на 
участках, идущих над прямой линией, компенсируется 
суммой площадей между прямой и кривой линией на тех 
участках, где кривая была ниже прямой линии). 

Ещё раз: в "мёртвой" точке развивается далеко не 
минимальная сила, потому как тут сила наиболее близка 
к статической (если не считать стартовой точки), и 
даже то, что она развивается в неудобном месте, не 
компенсирует роста силы за счёт практически полной  
остановки штанги. Действительно же минимальная сила 
развивается при жиме на этапе до "мёртвой" точки, хотя 
статическая сила в этих точках и выше, чем в самой 
"мертвой" точке, но из-за относительно высокой 
скорости движения сила тут падает ниже, чем в самой 
"мёртвой" точке, которую штанга минует на очень 
небольшой скорости.  

Я на этот момент особо упирал в одном из ответов 
Составителю - так что, уважаемый Чумбурмучка, будьте 
внимательнее.
 
Ответить	Открыть подтему

Тема: Ещё раз о "мёртвых" точках
Автор: Чумбурмучка
Дата: 08/04/2004 16:32
 	
Буду.
 
Ответить	Открыть подтему

Тема: Вот, нашёл наглядное пособие...
Автор: VadimPro
Дата: 08/04/2004 17:03
 	
Уважаемые участники дискуссии, для того, чтобы Вы 
могли более отчетливо представить себе ситуацию со 
средними силами, я предлагаю Вам взглянуть на данный 
график:

http://www.olifting.narod.ru/chemerkinclean260/chemerki
ncleangraphs.html

Красным цветом тут обозначена сила атлета, 
прикладываемая к штанге во время целого 
тяжелоатлетического движения. При этом в начале 
движения штанга стоит на помосте, а в конце движения 
она покоится на плечах атлета. В соответствии с 
провозглашённым мной принципом, и в этом случае 
средняя сила, прикладываемая к штанге, тоже должна 
быть равна весу штанги. 

Посмотрите на график и проведите прямую линию от 
стартовой до финальной точек - эта линия и есть 
средняя сила атлета, прикладываемая к штанге и равная 
её весу. Как можно убедится, площади между кривой 
линией силы и осью х равны площади между осью Х и 
линией средней силы. Пики силы над линией средней 
силы полностью компенсируются провалами силы в других 
местах. То есть, несмотря на то, что имеются участки 
где сила атлета значительно превышала вес штанги, были 
и такие участки, где атлет почти не прикладывал к 
штанге силу (во время уходов под штангу). И в 
результате итоговая средняя сила всё равно равна весу 
штанги. 

Всё то же самое имеет место быть и при жиме штанги -
если в конце движения штанга имеет минимальную 
скорость. Только тут средняя сила имеет гораздо больше 
физического смысла, чем при толчке, так как тут не 
будет резких возвышений и провалов силы, а будут 
небольшие отклонения силы от веса штанги, как выше 
линии средней силы, так и ниже её.


[на главную страницу]

Архив переписки

Форум



Спорт глазами Мовлади Абдулаева, тренера 
тяжелоатлетов

Weightlifting database

Weightlifting database

Мир тяжелой атлетики

Железный мир

Силовые виды спорта в Твери и в Тверской области

Тяжёлая атлетика глазами Артура Шидловского

Старые силовые Журналы США

Силачи прошлого и настоящего

Пауэрлифтинг от Петра Кравцова

Библиотека материалиста

Извлечённое из интернета